6R Solutions

February 11, 2018 | Author: rajatgoyal20 | Category: Comet, Violin, Gases, Redox, Voting
Share Embed Donate


Short Description

6R Solutions...

Description

MCAT Practice Test 6R Solutions ®

Solutions

6R

www.PrincetonReview.com

MCAT

MEDICAL COLLEGE ADMISSION TEST

1

MCAT P RACTICE T EST 6R S OLUTIONS Edited, produced, typeset, and illustrated by Steven A. Leduc National Director of MCAT Research, Production & Development, The Princeton Review

Jennifer Wooddell Verbal Reasoning Solutions Steven A. Leduc Physics and General Chemistry Solutions Judene Wright Biology Solutions Douglas S. Daniels Organic Chemistry Solutions

Copyright © 2003, 2002 by Princeton Review, Inc. All rights reserved. MCAT is a service mark of the Association of American Medical Colleges (AAMC). TPR is not affiliated with Princeton University or with the AAMC. Version 1.0

www.PrincetonReview.com

MCAT P RACTICE T EST 6R S OLUTIONS C ONTENTS : Physical Sciences ............. 3 Verbal Reasoning .............. 12

Biological Sciences ....... 30

030316

2

PHYSICAL SCIENCES Passage I 1. D. Which of the four compounds listed in the choices would produce a basic (alkaline) solution in a lake? Since KOH (choice D) is a strong base, its dissolution in water would produce a basic solution, so this is the best choice. Choice C, sulfuric acid, would produce an acidic solution, while neither choice A nor B would affect the pH. 2. C. A hydrogen bond is the intermolecular force between the partial positive charge on an H bonded to an F, O, or N atom and the partial negative charge on an F, O, or N atom in another molecule. Methane (CH4) has no F, O, or N atoms, and carbon dioxide (CO2) has no H atoms, so if we’re asked for the pair of compounds that can form extensive networks of hydrogen bonds with both participating, we must eliminate choices A and B (which involve methane) and choice D (which involves carbon dioxide). 3. D. Isotopes of an element differ only in the number of neutrons they contain, not the number of protons; this eliminates choices A and B right away. Carbon-12 has 6 protons and 6 neutrons, while carbon-14 has 6 protons and 8 neutrons. Therefore, these isotopes differ by 8 – 6 = 2 neutrons, choice D. 4. C. Since HCl is a strong acid, it will dissociate completely in solution. So, if [HCl] = 0.01 M, then [H+] = 0.01 M, and pH = –log [H+] = –log (0.01) = –log (10–2) = –(–2) = 2. 5. D. Here are the geometric families and shapes of the given molecule, methane, as well as those of the four choices: H C

H

H H

S H

H

O C O

CH4 geometric family: tetrahedral shape: tetrahedral

H 2S geometric family: tetrahedral shape: bent

F

F Xe F

F

Cl

CO 2 geometric family: linear shape: linear

Si

Cl

Cl Cl

XeF4 geometric family: octahedral shape: square planar

SiCl 4 geometric family: tetrahedral shape: tetrahedral

While both H2S and SiCl4 are in the same geometric family as methane, H2S has a bent shape while SiCl4 has a tetrahedral shape. Therefore, of the choices given, SiCl4 (choice D) is most like CH4, and thus the best choice here. 6. B. In a very low pH environment, the amino group of glycine, H2N–CH2–COOH, is protonated (as is the carboxyl group), giving H3N+–CH2–COOH, choice B. 7. A. Helium (choice B) is an inert gas and does not accept electrons. H2 (choice C) is a stable molecule and is therefore unlikely to accept electrons. Iron (choice D) is a metal and is therefore far more likely to donate electrons (becoming a cation) than to accept them. However, sulfur (choice A) is in the oxygen family (“Like oxygen atoms . . .”) and has a high electron affinity, so it’s the best choice here.

3

Passage II Note: The questions in this passage are on periodic trends or atomic structure. The passage text is irrelevant. 8. A. Atomic radius generally decreases as we move down a column or across a row to the right in the periodic table. The four atoms listed as choices are all in the same row (Period 3), and sodium (choice A) is farthest to the left, so we would predict that it has the largest radius of the four. 9. D. Ionization energy generally increases as we move up a column or across a row to the right in the periodic table. The four atoms listed as choices are all in the same row (Period 4), and krypton (choice D) is farthest to the right, so we would predict that it has the highest ionization energy of the four. Furthermore, since krypton is a noble gas, it has the particularly stable octet configuration and thus a very high ionization energy. 10. C. Since strontium (Sr) has atomic number 38, a (neutral) strontium atom has 38 electrons. Since the total number of protons and neutrons in strontium-90 is 90, the sum of the numbers of protons, neutrons, and electrons is 90 + 38 = 128. 11. B. Bromine (Br) has atomic number 35, which means that every bromine atom contains 35 protons. If an isotope of bromine has 44 neutrons, then the total number of protons and neutrons—which is the mass number—is 35 + 44 = 79. 12. C. Electronegativity generally increases as we move across a row to the right in the periodic table. As a result, bonds between atoms on the far left with those on the far right (excluding those in Group 18, the noble gases) tend to be highly polar (ionic). Of the four pairs listed as choices, Ca (calcium) and I (iodine) are farthest apart—Ca is in Group 2, I in Group 17—and would therefore be the most likely to form an ionic bond.

Passage III 13. B. Using distance = rate ¥ time, we get d = (3 ¥ 108 m/s) ¥ (7 ¥ 10–2 s) = 21 ¥ 106 m = 2.1 ¥ 107 m. 14. D. Any object moving in a circular path must feel a force that provides the centripetal force (choice D). For a satellite orbiting the earth, it is Earth’s gravitational force that provides the necessary centripetal force. Choice A is wrong because centrifugal force is a fictitious force directed away from the center of the circle and does not keep an object moving in a circular path. Choice B can be eliminated since, for one thing, at the altitude of the GPS satellites, there is no atmosphere. And we can eliminate choice C, since the earth’s gravitational force on the satellite is much stronger than the moon’s (so it doesn’t offset it). 15. A. This is a proportion question using Newton’s Law of Gravitation, F = GMm/r2. If we increase r by a factor of 6 and m by a factor of 4, the ratio m/r2 will change by a factor of 4/62 = 1/9, so F will decrease by a factor of 9: F¢ = G

M (4 m) 4 Mm 1 Mm 1 ◊G 2 = ◊G 2 = ◊ F 2 = 36 9 9 (6r ) r r

16. C. The passage tells us that the voltage of the Ni–Cd battery is 1.32 V. Since we know V and we’re given P (the power), we use the equation P = IV to determine the current: I = P/V = (3.96 W)/(1.32 V) = 3 A. 17. C. The last sentence of the passage tells us the two frequencies that are used: f1 = 102.1 MHz and f2 = 104.9 MHz. By definition, the beat frequency is the difference between the two: fbeat = f2 – f1 = 2.8 MHz = 2.8 ¥ 106 Hz. 18. D. The speed of an electromagnetic wave through a medium is given by the equation v = c/n, where n is the medium’s index of refraction. The vacuum of space has n = 1, so v = c; however, through any material medium (such as atmospheric gases), n is greater than 1, so v will be less than c.

4

19. A. We’re told in the first line of the second paragraph of the passage that the radioactive cesium source has a mass of 5 grams. So, the question is, How many half-lives must elapse to get this mass down to 0.625 grams? Dividing 5 grams by 2 repeatedly until we reach 0.625 grams, we see that it takes 3 half-lives: 1 half-life

2nd half-life

3rd half-life

5 g æ æææÆ 2.5 g æ æææ æÆ 1.25 g æ æææ æÆ 0.625 g Since each half-life is 175 years, the time period required would be 3 ¥ (175 years) = 525 years.

Passage IV 20. C. The second sentence of the passage states that the “synthesis of Olestra starts with a base-catalyzed cleavage . . .” (emphasis added). Of the choices given, only choice C, NaOH, is a base. 21. C. Since the specific heat of water is 1 cal/g∞C—or, equivalently, 1 kcal/kg∞C—the fact that the temperature of 1 kg of water increased by 50∞C means that the energy added to the water was 50 kcal: Ê 1 kcal ˆ q = mcDT = (1 kg)Á ˜ (50∞C) = 50 kcal Ë kg ◊ ∞C ¯ So, if the combustion of 10 peanuts releases 50 kcal of energy, the combustion of 1 peanut would release 50/10 = 5 kcal. 22. A. In the last sentence of the third paragraph of the passage, we learn that “Olestra is not metabolized. . . .” If it’s not metabolized, it won’t contribute any dietary calories to a human consumer. 23. A. From the structure of glycerine shown in Figure 1 in the passage, we see that this molecule contains 3 –OH groups, while the formula given in the question tells us that isopropyl alcohol contains just 1 –OH group. Therefore, not only does glycerine have a greater molecular weight, but it will have far more hydrogen bonds than isopropyl alcohol. As a result, the boiling point of glycerine should be much higher than that of isopropyl alcohol (choices C and D are eliminated, and A is a better choice than B). [By the way, the BP of glycerine is 290∞C, while the BP of isopropyl alcohol is just 82.5∞C.] 24. B. First, eliminate choices C and D. A mere 20 moles of methanol will not weigh several hundred thousand (and certainly not more than a million) pounds! The molecular weight of methanol, CH3OH, is 12 + 3·1 + 16 + 1 = 32 g/mol. Therefore, 20 moles of CH3OH has a mass of (20 mol)(32 g/mol) = 640 g, or 0.64 kg. Since the last sentence of the passage tells us that 1 kg “equals” 2.2 pounds, we find that 0.64 kg is equivalent to (0.64 kg)(2.2 lb/kg) ª (0.7)(2) = 1.4 lb.

Independent Questions 25. B. Electron affinity generally increases as we move to the right across a row or up within a column in the periodic table. Of the atoms listed as choices, fluorine (choice B) is farthest to the right and highest in the periodic table, so we’d expect that it would have the highest electron affinity. (Furthermore, if F acquires an extra electron, it achieves the much-desired noble gas (octet) configuration, so F “really wants” an extra electron; therefore, it has a high electron affinity.) 26. C. The half-cell reduction reaction of copper is shown directly in the table, Cu+ + e– Æ Cu, with E∞ = +0.52 V. If copper is to be reduced, then the other metal used in the cell must be oxidized, and we must remember that to form a galvanic cell, we need the overall cell voltage to be positive. The oxidation of silver has a potential of –0.80 V (the reverse of the reduction of Ag+ given in the table), so the overall cell voltage here would be –0.28 V, and silver is eliminated from consideration (and so choices A and D are wrong). Note that we don’t need to check lead (because it’s in both of the remaining choices, B and C). Since the oxidation of zinc has a potential of +0.76 V (the reverse of the reduction of Zn2+ given in the table), the overall cell voltage here would be +1.38 V, so zinc will work. Therefore, the answer must be C. 27. A. Frequency ( f, the number of cycles per second) and period (T, the number of seconds per cycle) are reciprocals of each other. Since f = 1/T, we have f T = 1.

5

28. A. Use Big Five #2, and call down the positive direction. Then v0 = –v (it’s negative, because v0 is upward and we’re calling down the positive direction) and vfinal = +v, so vfinal = v0 + at becomes v = (–v) + gt, or 2v = gt, which gives t = 2v/g. This means that t is inversely proportional to g. So, if g is decreased by a factor of 6, then t will increase by a factor of 6.

Passage V 29. C. In the reaction 2 Ag + H2S Æ Ag2S + H2, the Ag displaced H from H2S and took its place, forming Ag2S. This is an example of a single replacement (or single displacement) reaction. 30. A. By the stoichiometry of Equation 1 (which is balanced), 2 moles of SO42– ions would produce 2 moles of H2S. Since the molar masses of H and S are 1 and 32, respectively, each mole of H2S has a mass of 2·1 + 32 = 34 g, so the mass of 2 moles of H2S is 2 · 34 g = 68 g. 31. B. Because H2S is a weak acid (as stated in the sentence above Equation 1 in the passage), its ionization constant, Ka, is small. Since most weak acids have Ka values in the range 10–12 to 10–2, we’d certainly expect that the Ka value of H2S is in this range as well, and thus be much less than 1. (Granted it’s small, so we could say that it’s “near zero,” but the fact that Ka values can actually be measured and used in calculations means that we wouldn’t say that all weak acids have ionization constants that are negligible (i.e., near zero); we’d instead simply say that they’re very small. So, A is a good choice, but B is a better choice here.) 32. B. Choices A and D are not reactions between the two compounds; instead, they show how the ions would react to form each compound separately; these choices can therefore be eliminated. Only choices B and C show the ion from one compound reacting with the opposite ion from the other compound. Choice B shows the reaction that forms “the insoluble CaCO3, which crystallize[s], encapsulating . . . coins, sand, and decaying matter into rock-like clumps . . . [which is what] the explorers who discovered the treasure [in the ocean near Cuba] found . . . ,” as mentioned in the passage. This is the best choice here. Choice C shows the formation of NaCl(s), which we know is soluble, especially since the passage talks about finding treasure in the ocean, where NaCl would be dissolved into its separate ions. 33. D. Let’s see how the oxidation numbers of the atoms change as a result of the reaction: oxidation numbers:

0

+1 –2

+1 –2

0

2 Ag(s) + H2S(aq) Æ Ag2S(s) + H2(g)

Since Ag was oxidized (its oxidation number increased), it acted as the reducing agent.

Passage VI 34. D. We learn in the second-to-last sentence of the first paragraph of the passage that tones separated by a perfect fifth differ in frequency by a factor of 2/3. This means that the higher-frequency tone has a frequency that is 3/2 times the lowerfrequency tone when the tones are separated by a perfect fifth. Since the equation given in the passage tells us that f is proportional to T1/2 (where T is the tension in the string), if we want to increase f by a factor of 3/2, then we have to increase T by a factor of (3/2)2, which is 9/4. 35. C. According to the equation given in the passage, f = (T/r)1/2/(2L), we can lower f by lowering the string’s tension T, increasing the string’s linear mass density r, or increasing the string’s length L. Of the choices given, only C (using heavier strings—that is, strings with a greater linear mass density r) corresponds to one of these options. 36. D. In the last paragraph, the passage states that “the best violins produce loud tones over the full frequency range of the instrument . . .” (emphasis added). Each string has a characteristic fundamental frequency, so in order to produce loud tones over the instrument’s full frequency range, we’d want the violin to have good resonance at the fundamental frequencies of all the strings.

6

37. B. In the second paragraph of the passage, we learn that the A string of the violin or viola has a frequency of 440 Hz, and that “the cello is tuned one octave below the viola, which means the frequencies of the cello strings are half that of the viola strings.” So, if the A string of the viola is tuned to 440 Hz, the A string of the cello is tuned to 12 (440 Hz) = 220 Hz. 38. D. In the second paragraph of the passage, we learn that the A string of the violin has a frequency of 440 Hz, so a list of the four fundamental frequencies of the violin must include 440 Hz. This eliminates choices A and B. We also learn that the violin’s strings are tuned with decreasing frequency to the notes E, A, D, G. So, if A is 440 Hz, then E must be greater than 440 Hz. Only choice D includes a frequency higher than 440 Hz.

Passage VII 39. B. The atom is attached to 2 springs, each of force constant K. When an atom displaced a distance x from its equilibrium position, each spring exerts a force of magnitude K x , so the magnitude of the total force on the atom is 2K x . This means that the effective spring constant for this system is 2K. 40. D. Frequency ( f, the number of cycles per second) and period (T, the number of seconds per cycle) are reciprocals—or inverses—of each other. 41. C. The question tells us to use a “dimensional argument” to identify the correct formula for T in terms of K and M. This means that all we need to check is that the dimensions (or units) of the two sides of the equation match. The left-hand side of each equation in the answer choices has units of seconds2, since the period T is measured in seconds and p is a constant. Therefore, we need to determine which right-hand side also has units of seconds2. From the equation F = K x , we know that the units of K are N/m, which is the same as kg/s2. So, to get s2 out of this, we need to flip K—to get s2/kg— and then multiply by kg (mass). Therefore, the right-hand side of the equation must contain the term (1/K)·M, or M/K. Only choice C contains this term. 42. A. For an oscillatory system without dissipative effects due to friction, the total mechanical energy, E, is conserved. As the oscillator moves, there is a continuous trade-off between kinetic and potential energy. The kinetic energy and the potential change individually, but their sum, E = KE + PE, remains constant (choice A). Note that momentum is constantly changing since the speed and direction of the oscillator are constantly changing. 43. D. In the first line of the second paragraph of the passage, we’re given the formula for the potential energy of a spring: PE = Kx2/2. Since there are 2 springs here, the potential energy will be Kx2/2 + Kx2/2 = Kx2. When the atom is at its greatest distance from equilibrium—that is, when x = ±A, where A is the amplitude—there is no kinetic energy, only potential energy. Therefore, E = KE + PE = 0 + KA2 = KA2. Solving the equation E = KA2 for K, we get K = E/A2 (choice D). We could have also answered this question by just using a dimensional argument, like in Question 41 above. Since [E] = J = N·m, and [K] = N/m, we’d need to divide N·m by m2 to get N/m. That is, we’d have to divide E by A2 to get K, so the answer must be D. 44. B. From the equation q = mcDT, where q is heat (energy), m is mass, c is specific heat, and DT is the temperature change, we find that c = q/(mDT), so [c ] =

[q ] J = [ m ][ DT ] kg ◊ K

45. A. The first sentences of the passage say, “let us assume that each atom can oscillate about its equilibrium position. Interactions with neighboring atoms hold it in place. . . .” Therefore, the model described is limited by this assumption, which holds very well for solids. However, we know that the particles of a monatomic gas move very freely; they are not restricted to small oscillations around fixed equilibrium positions established by interactions with neighboring particles. (In fact, in the kinetic–molecular theory of ideal gases, we assume that there are no such interactions.) Since the basic assumption of the model fails for gases, it can’t be used to describe gases.

7

Passage VIII 46. B. Compound 1 contains a four-membered ring, while Compound 2 contains a six-membered ring, so their structural formulas are clearly different. Also, Compound 1 contains 2 B’s, 2 P’s, 2 H’s, and 2 NR2 groups, while Compound 2 contains 3 B’s, 3 P’s, 3 H’s, and 3 NR2 groups, so their molecular formulas are different, too: B2P2H2(NR2)2 vs. B3P3H3(NR2)3. However, their empirical formulas—which give the smallest whole numbers that specify the correct ratios of the atoms—are the same: BPHNR2. 47. C. From the stoichiometry of Equation 1a (which we assume is balanced), the given initial amounts of reactants will produce 0.003 mol of PH3(g). Since the volume of 1 mole of gas at STP (0∞C and 1 atm) is 22.4 L, the volume of 0.003 mol of gas at STP will be (0.003)(22.4 L) = (3 ¥ 10–3)(22.4 L) = (3 ¥ 22.4)(10–3 L) = 67.2 mL. 48. A. Choices B and C are identical, so both can be eliminated. Now, since a phosphorus (P) atom has 5 valence electrons, not 7, the answer must be A, not D. 49. A. From the Ideal-Gas law, PV = nRT, we know that V = (nR/P)T, so a graph of V vs. T should be a straight line with slope nR/P, which is always positive. (This eliminates choice D, by the way.) If the pressure P is a constant, then the slope is proportional to n, the number of moles in the sample. Both samples have a mass of 1 gram, but Compound 2 has a greater molar mass than Compound 1 (1.5 times as much, to be specific). Therefore, the value of n for the Compound-2 sample is lower than the value of n for the Compound-1 sample. As a result, the V vs. T graph for Compound 2 will have a smaller slope than the one shown for Compound 1, and would thus appear below the plot for Compound 1. [Note: The graph of P vs. T for any ideal gas is a straight line that would intersect the T axis at a common point: –273.15 ∞C; that is, at absolute zero (0 K). For the graphic shown with the question, the unit on the T axis is kelvins, so the lines would all intersect the T axis at the origin. In either case, a line with a smaller slope must appear below a line with a greater slope.] P

P

greater slope

greater slope

smaller slope

–273.15

smaller slope

0

T (∞C)

T (K)

Independent Questions 50. A. The term isoelectronic means same number of electrons. Because a neon atom has 10 electrons, the question is asking for the atom or ion that doesn’t have 10 electrons. The answer is A, helium (since it has only 2 electrons). 51. D. The work W required to lift an object of mass m through a vertical distance h in a uniform gravitational field is given by the expression mgh, the product of the gravitational force, mg (which we’d have to provide to lift the object) and the distance, h. Therefore, W is proportional to h, which implies that if h is doubled, then so is W. 52. A. By Graham’s law of effusion, we know that lighter gases diffuse faster than heavier ones. Because Gas X has a lower density than Gas Y (at, we assume, the same pressure and temperature), the molecules of Gas X must be less massive than those of Gas Y. Therefore, Gas X will diffuse faster, because it has a lower molar mass than Gas Y. 53. B. A catalyst has no effect on a reaction at equilibrium, so the amount of ammonia at equilibrium with a catalyst will be the same as the amount of ammonia at equilibrium without a catalyst. 54. D. A body is in translational equilibrium if the sum of the external forces on it is zero. A body is in rotational equilibrium if the sum of the external torques on it is zero. (And a body is said to be simply in equilibrium if it’s in both translational and rotational equilibrium.)

8

Passage IX 55. C. Imagine a rocket floating in free space. If its engines are ignited, causing chemical reactions that expel exhaust gases, the rocket will accelerate in the opposite direction. We can view this as a consequence of Conservation of Momentum. The system originally had zero momentum; if part of the system then moves in one direction, the other part of the system must move in the opposite direction so that these two oppositely-directed momenta add up to zero. But we can also view this as a result of Newton’s Third law (“to every action, there is an equal but opposite reaction”). The rocket exerts a force expelling the gases, and the gases thus exert a reaction force, in the opposite direction, on the rocket. The same is true for a comet expelling gas as a result of sublimation of some of its frozen solids, so choice C is the best choice. Choice A can be eliminated since Newton’s First law tells us what happens in the absence of a net force; it wouldn’t account for the production of a force. Choice B can also be eliminated; Newton’s Second law will tell us how the comet will react to any force (including the perturbing force), but it doesn’t account for the perturbing force. And, finally, choice D is eliminated since the third sentence of the second paragraph of the passage specifically tells us that the deviations in the orbits of comets (caused by the perturbing force) are nongravitational. 56. C. The passage states that the sublimation of the frozen solids in a comet’s nucleus is caused by sunlight warming its surface. As the comet orbits the sun, only the portion of the surface facing the sun receives sunlight. As a result, gases are ejected from the side of the nucleus facing the sun, which causes the reaction force—which acts in the direction opposite to the direction of the expelled gases— to be exerted on the nucleus in the direction away from the sun.

comet nucleus reaction force (away from the sun)

57. B. According to a typical phase diagram, solids can sublime only in conditions of low pressure (specifically, below the pressure at the triple point). Therefore, choice B is best.

SUN

gases

P solid vapor T

58. A. Choices B, C, and D are all false, so the answer must be A. A (frozen) solid is held together by relatively strong intermolecular forces. A pile of rocks, on the other hand, would be held together only by the gravitational force, which is much weaker. So, when the “rock pile” comet strikes the gas molecules in the atmosphere of Jupiter, the collisions will be much more effective at breaking up a rock-pile nucleus than they would be if the nucleus were a single solid piece (of frozen compounds). 59. B. As stated in the last paragraph of the passage, “Comets become visible to the unaided eye when . . . the coma forms and extends into a vast, long tail of gas and dust.” We therefore eliminate choice D. (While ices can reflect light, they’re in the nucleus, which is concealed by the surrounding cloud of dust particles and gas, as stated in the second sentence of the first paragraph of the passage.) So, how do we choose among choices A, B, and C? One way is to realize that two of these choices mention gas, while only one mentions dust. Since there can’t be two correct answers, we’d be led to eliminate the two that mention gas, and thus select choice B. Another way is to realize that diffuse gases are very poor reflectors of light (for example, we don’t wear sunglasses because sunlight is reflecting off air molecules; we wear them to help shield our eyes from the reflection of sunlight from solids and liquids, not gases). This observation eliminates choices A and C, so choice B is indeed the best. 60. C. The two models differ on the question of what the cometary nucleus is made of—is it made of rocks or is it a solid of frozen ices? If we were able to perform a spectroscopic study of the cometary nucleus, we’d be able to determine the composition of the nucleus and thus decide between the two models.

9

Passage X 61. C. From Equation 1, PbI2(s) Pb2+(aq) + 2 I–(aq), we have Ksp = [Pb2+][I–]2. If S moles of PbI2 dissolve in 1 liter of water, then we’ll have [Pb2+] = S and [I–] = 2S. Substituting these into the equation for Ksp gives Ksp = (S)(2S)2 = 4S3. 62. D. By analogy with Equation 1, the dissociation of lead(II) bromide is PbBr2(s) Pb2+(aq) + 2 Br–(aq), so Ksp = 2+ – 2 2+ – 2+ [Pb ][Br ] . With the given initial concentrations of Pb and Br , [Pb ] = 0.0001 M = 10–4 M and [Br–] = 0.00005 M = 5 ¥ 10–5 M, the initial reaction quotient (or ion product, in this case) is [Pb2+][Br–]2 = (10–4)(5 ¥ 10–5)2 = 2.5 ¥ 10–13. Since this is smaller than the Ksp of lead(II) bromide—given to be 4.6 ¥ 10–6 in the last sentence of the passage—there will be no precipitate. Choice D is the answer. 63. B. As given in the last sentence of the paragraph that follows Equation 2 in the passage, the molar solubility of PbI2(s) in water is S = 1.89 ¥ 10–3 M. So, in 100 mL = 0.1 L of a saturated aqueous solution of PbI2, there’ll be (0.1 L)(1.89 ¥ 10–3 M) = 1.89 ¥ 10–4 mol of dissolved lead(II) bromide. Since the molecular weight of PbI2 is 207 + 2(127) = 461 g/mol, the mass of PbI2 present in the saturated solution is (461 mol )(1.89 ¥ 10 -4 mol) ª (450 g

g -4 mol )(2 ¥ 10

mol) = 900 ¥ 10 -4 g = 0.09 g

Therefore, choice B is best. 64. A. Because the value of Keq (the ratio of “products to reactants” at equilibrium) for Equation 2, given to be 5 ¥ 1015, is so large, the reaction goes essentially to completion. That is, we’d say that essentially all (100%) of the reactant I–(aq) is consumed—in this case, converted to I2(aq). 65. D. As shown in the solution to Question 62 above, the expression for the Ksp of lead(II) bromide is [Pb2+][Br–]2.

Passage XI 66. D. The equation F = QE for the electric force on a charge Q says that if Q is positive—as it is in this experiment, since (as stated in the second sentence of the first paragraph of the passage) “atoms are ionized by removing electrons”—then F is in the same direction as the electric field, E. In order to accelerate an object to the right, we need a force F directed to the right, and, therefore—since the objects each carry a positive charge—an electric field E that’s directed to the right. 67. C. The superscript 6 in 6Li is the mass number of the atom, which, by definition, is the number of protons plus the number of neutrons. (Note that choice A can be eliminated immediately, since a glance back at the periodic table reveals that every lithium (Li) atom contains exactly 3 protons, not 6.) 68. A. From the equation given in the passage, we find that v = 2QV M . So, if V is decreased, the speed v of every ion will decrease. Since the ions are all moving slower, they’ll all require more time to reach the detector. 69. C. Let x be the distance from the right-hand plate to the detector and let V be the voltage between the accelerating plates; these values are fixed. The time required for an ion to travel this distance (the “flight time”) is equal to t=

x = v

x 2QV M

=x

M 2QV

where we’ve used the expression for v given in the solution to Question 68 above. Since 3H+ and 3He+ have the same M and the same Q, the value of t will be the same for both ions. 70. D. Because “Peaks 3 and 4 are the peaks . . . for . . . singly-ionized isotopes of lithium” (emphasis added), as stated in the last sentence of the passage, the peaks for doubly-ionized lithium isotopes won’t include Peak 3 or Peak 4. This observation eliminates choices A, B, and C, so the answer must be D. [Alternatively, since t = x M 2QV , a greater Q leads to a shorter flight time. Therefore, doubly-ionized lithium ions will have shorter flight times than singly-ionized lithium ions. So, if Peaks 3 and 4 correspond to singly-ionized lithium, the peaks for doubly-ionized lithium will appear at shorter flight times. Only Peaks 1 and 2 in Figure 2 are at shorter flight times than Peaks 3 and 4.] 10

71. A. Peak 5 corresponds to a much greater flight time than Peak 4, which we know is the peak for singly-ionized 7Li. Since t = x M 2QV , we can say that t is proportional to M , because x and V are fixed and Q is a constant here (equal to +1e). So, if t is greater for Peak 5 than for Peak 4, Peak 5 must correspond to a species with a greater mass, M, than 7Li. This eliminates choice D. While choices B and C might be true (if they lead to an overall greater mass for the ion), choice A would certainly lead to a greater M, so it’s the best choice here.

Independent Questions 72. B. Since HCl is a strong acid and NaOH is a strong base, the question is asking which of the substances listed as choices would be more soluble in an acidic solution than in a basic one. Bases are more soluble in acidic solution than acids are (a consequence of Le Châtelier’s principle or the common-ion effect), so choice B—Pb(OH)2, the only base listed—would be more soluble in 1 M HCl than in 1 M NaOH. Choice D, HI (a strong acid), would be more soluble in the NaOH solution; choice C, CaF2, has no apparent preference for either solution; and, although it has no acidic or basic properties in water, AgCl (choice A) should be more soluble in the NaOH solution than in the HCl solution due to the common ion effect (Cl– from the AgCl and Cl– from the HCl). 73. C. Choices A, B, and D are all false: Sounds travels faster through solids than through gases, the frequency of the sound wave would be the same as it passes through the wall, and the wavelength of sound is longer in a solid than in a gas (because l = v/f, and v is greater in a solid and f is the same). Therefore, the answer must be C. If part of the sound energy is reflected off the wall (and it is), there’ll be less to pass through the wall to the listener on the other side, and less energy will mean less intensity. 74. B. Hooke’s law can be written as DL = FL/AY. If the second wire is made of the same material as the first, it has the same Young’s modulus, Y. If the same mass M is used, then F (which is Mg) is the same. Therefore, DL is proportional to L/A. So, if L is doubled and the radius of the wire is doubled (meaning the cross-sectional area A is quadrupled), then the ratio L/A will change by a factor of 2/4 = 1/2; that is, DL will decrease by a factor of 2. More explicitly, we have the following calculation: DL ¢ =

2 FL 2 FL 1 FL 1 FL ¢ FL ¢ F (2L) = = = = = = DL A ¢Y pr ¢ 2Y p(2r ) 2 Y 4 pr 2Y 4 AY 2 AY 2

75. B. According to Faraday’s Law of Electrolysis, the amount of chemical change is proportional to the amount of electricity that flows through the cell. Therefore, the rate at which pure copper is produced in this electrolytic cell will be increased most effectively by increasing the electrical current through the cell. 76. D. Consider a column of fluid. We know that the fluid pressure near the base of the column is greater than near the top, because greater depth leads to greater fluid pressure. In the example described in the question, since the leg is lower than the arm, we’d expect the fluid pressure in the leg to be greater than in the arm. 77. D. The electric-circuit equation P = IV tells us that for a given P, the quantities I and V are inversely proportional. So, if the electric power P for transmission is “stepped up” to a very high voltage—that is, if V is increased—then the current, I, will decrease (eliminating choices A and B). As a result, the joule heat generated in the transmission wires, I2R, will decrease (because I decreases), by a lot (because I is squared). For example, if V is stepped up by a factor of 100, then I will decrease by a factor of 100, and the heat loss decreases by a factor of 1002 = 10,000. This reduction in the heat loss in the transmission wires means more useful electrical power is transported, and choice D is best. Choice C is eliminated, since a higher voltage would actually call for more insulation around the transmission wires to prevent accidental electric shock.

11

VERBAL REASONING Passage I 78. C A: The author’s thesis or main point is that the late 19th century British Mediterranean fleet sacrificed function for appearances, as shown by the “cult of brightwork” (line 20). The reference to doors in lines 14-17 shows that the crews cared about how well the doors were polished rather than about how well they worked. B: The fact that ammunition was often dumped overboard because firing the guns dirtied the ship (lines 61-63) supports the author’s thesis that the captain and crew, for the most part, cared more about the appearance of the ship than they did about how well the ship and crew could perform. C: Yes. The correct answer will be the one that least strengthens or supports the author’s main argument that the British Mediterranean fleet in the late 1800s became obsessed with appearances and neglected practical or functional issues. The author does not state or suggest that coal-loading competitions (lines 9-10) took place at the expense of more practical issues like gunnery practice or vermin control. D: This reference in paragraph four (lines 37-40) shows that the commander’s priority was how shiny the latrines (bathrooms) were, rather than how useful they were to the crew. Thus, it does support the author’s argument about putting appearances above function. 79. B A: The correct answer will be the one that is best supported by both the passage and the new information in the question. The passage never discusses respect for the British sailing tradition, and so gives no support for the idea that respect for tradition would have led officers to choose sails over engines. B: Yes. In the last two paragraphs, the author describes how commanders avoided using the ship’s guns because they didn’t want to get things dirty. From this we can reasonably infer that given the choice between sails and engines in a hybrid ship, commanders would choose the cleanest option and avoid the use of the engines. C: The author indicates in lines 21-24 that the sailors did in fact spend a great deal of time caring for the rigging on sailing ships, and that crews competed in their handling of the sails (lines 7-8). Therefore, there is no reason to believe that they lacked training in that area. D: Note that this choice is the opposite of choice A. In discussing the transition from sails to engines (lines 7-10 and 2124), the author never indicates whether or not commanders and crews had a preference for one technology over the other. 80. C Item I:

Item II:

Item III:

Yes. The author describes the lack of interest in gunnery practice in order to support his argument that commanders were obsessed with appearances. Furthermore, this obsession compromised their ability to carry out the real function of a navy ship, that is, to be able to fire its guns in defense of the nation (paragraphs five and six). Attempts to improve preparedness would then likely focus on gunnery practice, and these attempts would likely meet with opposition (lines 43-45). Yes. The author’s thesis, in general terms, is that late 19th century British commanders in the Mediterranean fleet emphasized appearances over function. Therefore, reform attempts would likely do the reverse, and prioritize function [e.g., watertight doors (lines 14-17), rodent control (lines 40-42), and the use of the ship’s guns (lines 43-46 and 55-62)] over appearances. The fact that the appearance of the ship [and of the admirals (line 5)] was so important does not support the inference that ceremony was important, or problematic, as well. Even if we imagine that the admirals’ visits to local dignitaries were ceremonial, the author does not suggest that these visits compromised the functioning of the ship. Secondly, while the “cult of brightwork originated in the need to keep the men busy” (lines 20-21), it is the cult—that is, the obsession with appearances—which is the problem in need of reform, not busywork itself. Finally, once you have eliminated answer choices based on passage support for items I and II, it is not possible for III to be correct, as there is no choice including all three numerals.

12

81. A A: Yes. The author suggests that the splendiferous appearance of the ships represented the “majesty and power of England” (lines 2-5). This would support an explanation based on politics, that is, the political power and status of England in the international arena. Be sure to use process of elimination aggressively. No other answer choice is both relevant to and consistent with information provided in the passage. B: According to the passage, these practices arose largely from the desire to embody the power of England, the competitive spirit between ships and commanders, and the need the keep crews busy. The reference to “Her Majesty’s ships” in line 32 is not enough to suggest that the queen personally knew or cared about the “cult of brightwork,” or that its practices were the result of direct royal commandments. C: According to the passage, the captains or commanders, not the admirals, bore most of the expense involved in keeping the ship shiny (lines 27-33 and 47-50). The passage does state that the admiralty understood the problem caused by dirty gunsmoke and that it looked the other way when commanders avoided the problem by dumping the ammunition (lines 62-63). However, it does not indicate that the admirals themselves bore any of the costs of clean up when the guns were actually fired. Finally, the fact that the admiralty required gun drills to be held four times a year (lines 51-53) indicates that they were not trying to avoid expenses attached to those drills. D: The author suggests that the British navy’s appearance of power and majesty did not reflect reality, just as the bright and shiny appearance of an individual ship hid real problems in how well that ship could function (lines 14-17 and 3442). The lack of military or gunnery drills likely detracted from the British navy’s true military might, rather than being an indication of its military superiority. 82. A A: Yes. In answering this question, remember both the main idea of the paragraph in which the cited lines appear, and the main point of the entire passage. In this paragraph, the author begins the discussion of the commanders’ obsession with the ship’s appearance. Notice the wording in lines 17-19; by saying that “even the ring bolts” were polished and fitted with little hats, the author is indicating that these measures were taken to absurd lengths. B: The positive tone of this choice (especially the word “innovative”) is inconsistent with the critical tone of the paragraph and of the passage as a whole. Furthermore, the need for a “sparkling ship” is not described as a “distinctive challenge” to be met by the commanders, but as something that was largely created by the commanders themselves, fueled by the competitive spirit between them. C: This is the right answer to the wrong question. That is, it is supported by the passage, but it is not the specific point the author makes with this particular reference. It is especially tricky because the previous sentence regarding the gleaming but no longer watertight doors does indicate how appearance may interfere with function. However, there is no indication that the polish or little hats on the bolts interfered with their function, only that these were extreme and somewhat silly measures. D: This choice is both too positive in tone and too broad. The reference to polished ring bolts is not meant to represent meticulous naval standards as a whole, but ridiculously meticulous standards in the specific area of polished and clean surfaces. 83. C A: This choice is only partially correct. The crews were in fact inadequately trained in the use of the guns (see the last two paragraphs). However, the author gives no evidence of any expertise (beyond directing their crews in what and how to polish) on the part of officers. In fact, the author indicates that their skills lay largely in social and sporting events (lines 6-7 and 55-56). B: This choice is inconsistent with the main idea of the passage. The author, in particular by describing how gunnery practice was avoided whenever possible, indicates that the navy was unprepared for battle at the turn of the century. C: Yes. Gunnery lieutenants were the only ones pushing for target practice at the turn of the century (lines 53-55). Given that the crew’s ability to hit a target in battle is a major part of the function of a warship, it would be reasonable to infer that the ships’ success in World War II indicated that more target practice was held. D: Compare this choice to choice C. It is much too great a stretch to infer that British warships succeeded because the German ships were handicapped by the same counterproductive obsessions, given that there is no evidence regarding what the German navy was doing. It is much more reasonable to infer that the British training methods changed in response to complaints of gunnery officers, for which we do have some evidence in lines 53-55.

13

Passage II 84. B A: This is the right answer to the wrong question. That is, it is supported by the passage (lines 26-31), but is not described as an inherently negative thing, i.e., a drawback. Compare this choice to choice B. Reacting on the basis of personal feelings may or may not be a drawback; when it is, it is because those feelings do not fully respond to reality (lines 3638). Therefore, if A were correct, we would have two correct answers. B: Yes. See lines 36-38. C: Orientation is described as a benefit (lines 24-26 and 36-37), not as a drawback. D: The passage portrays partisanship as a long-term force that may be contravened by short-term forces (lines 39-53). Even when short-term forces strengthen partisanship (line 47), it is not because partisanship itself made voters more vulnerable to the influence of other factors. 85. A A: Yes. This is a paraphrase of lines 39-41. B: Measuring partisanship may help analysts know when short-term forces are at work (lines 39-53), but the author never suggests that it can help identify which specific short-term forces are involved. C: This contradicts the first sentence of the passage, in which party identification is described as an “uncomplicated measure.” Later in the passage, the author states that “party identification” and “partisanship” can be used interchangeably (lines 12-14). D: First of all, while partisanship may be related to a person’s childhood political experiences (lines 21-23), the author does not describe this as the basis for political analysts’ interest (lines 39-41). Secondly, this choice is too extreme to be supported by the passage even if it were relevant to the question. While a person’s party identification is usually acquired in childhood, the passage never indicates that the political climate during childhood is the only, or the most important, factor. 86. A A: Yes. The author lists “local circumstances” and “the attractiveness of the candidates” as short-term forces that may cause voters to “move away from their normal party choices” (lines 48-53). A local political scandal could make the officials involved less attractive to voters in their own party. B: There is no mention of political action committees or anything analogous to them in the author’s discussion of shortterm forces that may work against partisanship. C: The author suggests that a voter’s family may help form his or her long-term party identification (lines 21-24). He does not go on to suggest, however, that family members’ opinions would cause a voter to go against that party identity later in life. D: Campaign advertising, or any kind of propaganda from the opposing party, is never mentioned as a force that could contravene party identity. In fact, the author suggests that a voter’s reactions to such ads would be negatively affected by his or her existing party identification (lines 26-31). 87. C A: This choice is too extreme. The scenario in the question corresponds to the author’s discussion of short-term breaks in party loyalty (see the last two paragraphs). The author never discusses wholesale (that is, widespread and long-lasting) changes in partisanship. B: The author never discusses level of interest as a factor in unexpected electoral results. C: Yes. The author lists “attractiveness of the candidates” as one factor that could cause people to vote against their party identification (lines 48-50), and so that could lead to an unexpected electoral result (lines 60-62). Note the similarity in theme to Question 86, answer choice A. D: The passage indicates just the opposite. If domestic issues could affect the outcome of an election by spurring people to vote outside of their party (lines 50-53), the lack of any pressing domestic issues would leave voters with their longstanding preference, all other things being equal.

14

88. A A: Yes. The author argues that partisanship leads people to respond favorably to information from their own party (lines 26-34). B: The passage gives no reason to believe that disaffected voters, or those with a negative attitude toward the political process, would respond especially favorably to attack ads. C: The passage indicates just the opposite in lines 26-34. D: The passage indicates that people with a particular party identification, rather than those with no party identification, will tend to respond most favorably to ads that are from that party, or in that party’s interest (lines 26-34). 89. B A: The passage makes no comparison between partisans and nonpartisans on the basis of how much they care about politics as a whole. B: Yes. The author argues that partisanship can be useful to voters because it gives them a guide as to how to think about and react to information. Thus, if the information comes from their own party, they automatically tend to believe it (lines 24-36). So, it would make sense to say that a nonpartisan voter without this pre-existing guide might take longer to go through the same evaluative process, as more thought would be necessary. Be sure to use process of elimination aggressively. While this choice is a bit of a stretch, there is no support at all for the other three choices. C: No comparison is made on this basis. If anything, we would expect those without a long-standing party identity to be more, not less likely to make political decisions on the basis of short-term factors. D: This choice contradicts the main idea of the passage, which is in part that party identification, when it exists, exerts a powerful influence over voters’ political decision-making. If independents acted just like partisans, it would undermine the author’s argument. 90. D A: Notice the word “determined” in the question. Measuring the political predispositions of all voters is only one part of the process of measuring the effect of short-term forces (lines 55-62). B: Those views may relate to short-term forces at work in an election (lines 48-53), but measuring the views alone wouldn’t tell us the effect those views had on the election. C: As in choice B, the state of the economy may affect the election results, but measuring the state of the economy alone isn’t enough to tell us how big that effect was, according to the passage. D: Yes. In lines 60-62, the author states that the “departure from the expected [partisan] vote” indicates how much of an effect short-term forces had.

Passage III 91. A A: Yes. According to the passage, creationists see perfection as evidence of the work of a “divine architect” (lines 4146). Therefore, they would dislike the idea of rudimentary or useless biological structures, which would not provide good evidence for a “perfect original creator.” B: Creationists would like, not dislike, nonadaptive structures. Adaptation involves evolution or change over time in accordance with environmental pressures (lines 29-32). According to the author, creationists reject the idea of evolution, and so would be pleased by evidence of structures that have not changed to better fit with the environment (lines 41-46). C: As in choice B, this is the opposite of the correct answer. Creationists, according to the author, believe that biological structures have not changed since their moment of divine creation (lines 41-46). Therefore the idea of changeless structures would please, not displease creationists. D: We have no way of knowing from the information in the passage, in the question, or in this answer choice whether or not ornamental structures might serve some function (e.g., attracting mates), or if instead they might be useless appendages. Thus we have no way of knowing how creationists might feel about their existence.

15

92. B A: While the teeth may have served this function for the whale’s ancestors (lines 58-59), the author states that in the modern whale the teeth are useless and are resorbed (reabsorbed) at some point in time (lines 55-60). B: Yes. See lines 55-57. C: The temporary appearance of the teeth, as useless remnants from the past, is given as evidence of evolution that has already taken place, not of an ongoing evolutionary process (lines 47-60). D: These whales use a whalebone filter to eat, while the useless teeth are reabsorbed (lines 55-60). Teeth and filter are portrayed as separate, distinct structures. 93. D A: Make sure to read each word of the question carefully. Vestigial structures were once useful, but are now rudimentary and useless (lines 47-51 and 61-68). B: Adaptation occurs, according to the passage, when organisms evolve over time to better fit into their environment (lines 29-32). Vestigial structures, or features of an organism that are no longer functional, were once well adapted, that is, useful to the organism (lines 9-11 and 49-60). Thus we can infer that they are nonadaptive now, not in the past. As in choice A, be sure to pay close attention to the time reference (“once”) in the question. C: The passage suggests just the opposite. Vestigial structures are remnants of anatomical parts that once performed a function, but that are no longer useful. Thus they were mutable or changeable. D: Yes. The author and Darwin suggest that vestigial structures, as in the example of whale teeth described in lines 55-60, are leftover parts or useless versions of things that once performed a function but no longer do. Note also the words “become useless” in line 11. To become useless, an organ must once have been useful or functional. 94. C A: The author discusses the history of this word not to support a more general claim about language, but as an analogy used to support his argument about biological evolution. The question asks not only what choice is supported by the passage, but also what particular claim the discussion of linguistic change is primarily intended to support. B: The author does not make this claim. In fact, it is inconsistent with the author’s discussion of the value of historical study to science (see in particular lines 13-20). C: Yes. Here and in paragraph five the author uses the analogy of linguistic evolution to support his argument that vestigial biological structures must have had some function in the distant past. This argument becomes more clear in the second paragraph, which continues the same theme (lines 13-20). If an argument made in one paragraph continues on through other paragraphs, it may be useful or even necessary to look elsewhere in the passage for the answer to a line reference question. D: The author does in fact make this point (lines 9-12 and 52-55). However, changes in the meaning and pronunciation of the word “emolument” are mentioned not primarily to support a claim about language, but to support, by analogy, the author’s discussion of biological evolution. Noticing the word “primarily” in the question is the key to avoiding this “right answer, wrong question” trap. 95. D A: This is inconsistent with the author’s statement that the perfection of a gull’s wing could be seen as evidence of divine creation just as well as it could serve as evidence for evolution (lines 36-46). B: This choice is too extreme. While the author appears to support Darwin’s evolutionary argument, he never explicitly rejects or definitively denounces the creationists’ claim that perfection is evidence of the work of a “divine architect.” C: The author states that the perfect matching of organism to environment at a particular point in time does in fact exist, as shown by a variety of examples (lines 32-34), including that of the gull’s wing. D: Yes. The author states that Darwin looked to imperfection for evidence of evolution, as examples of perfection could be used to support either evolution or creationism (lines 35-46).

16

96. C A: The passage, following Darwin, suggests that vestigial structures (anatomical features that no longer serve important functions) provide the strongest evidence for evolution (lines 47-51 and 55-64). The highly developed brain exemplifies the opposite of a vestigial or rudimentary organ. B: An opposable thumb is a very useful appendage, while the best evidence for evolution, based on the passage, would be the existence of a useless structure (lines 47-51 and 55-64). C: Yes. If tonsils can be removed without compromising the body’s function, then they, like the appendix (lines 6-8), would qualify as vestigial. This would provide, according to the passage’s discussion of Darwin, evidence of human evolution (lines 47-51 and 55-64). D: An organ upon which life depends can hardly be called vestigial or rudimentary. It is to rudimentary or useless organs that Darwin looked for evidence of evolution (lines 47-51 and 55-64). 97. B A: Fossils would have no relevance to or impact on the author’s claim about linguistic evolution. The history of language is used to support the author’s argument about biological evolution, not the other way around. B: Yes. In the second paragraph, the author suggests that no direct evidence (“observational or experimental”) of evolution exists, and so that scientists must infer from characteristics of modern organisms that evolution occurred. Direct evidence in the form of fossils would significantly challenge this claim. C: Fossilized evidence of evolution would support, not undermine the claim that the world has changed over time. D: In the context of the passage and of the last paragraph, this assertion means that the origin of vestigial structures is simply or clearly the existence of a functional equivalent of that structure in the distant past (lines 64-68). Fossilized evidence of evolutionary processes would support, not undermine this assertion.

Passage IV 98. D A: Notice the word “normally” in the question. While the typical Oedipal path does lead a son to defy his father (lines 2930), it also leads the son to leave his mother and to marry a woman similar to her (lines 34-36). Victor Frankenstein, who according to the author does not fit the typical Oedipal model, identifies with his mother instead of relinquishing her (lines 63-67). Note the first line of the third paragraph, and the word “but” that begins the fourth paragraph. It is Homans, not the author who claims that Victor follows a typical developmental path. B: This is the path taken by Victor, who according to the passage reverses the normal Oedipal model. He leaves the symbolic or public realm in favor of a private, imaginary world represented by his mother’s “body” (lines 2-5 and 5053). C: While “patriarchal marriage” is mentioned in line 12, the author never presents patriarchy and matriarchy (matriarchy is never mentioned or described) as two systems between which young men must choose. This choice takes a word from the passage out of context. D: Yes. This is a paraphrase of lines 34-36. 99. C A: While the author of the passage describes Victor’s search for the magical powers of nature (lines 31-32 and 47-50), she does not suggest that the book presents an argument for those powers. The passage never indicates that Shelley believes we should choose a magical and mysterious nature over the public realm of society and official science, or that she holds up Victor as a model to be followed. B: See the explanation for choice A. While Victor leaves behind the patriarchal social realm (lines 11-13 and 50-53), the passage does not suggest that Shelley intended her book to be read as social criticism. Nor is it suggested that Shelley presented Victor’s rejection of a patriarchal society as a model to be followed by others. C: Yes. In lines 67-70, the author suggests that the story of the birth of a monster reflects Shelley’s own experiences. D: The passage never describes the novel or its author as skillful. Nor does it indicate that Shelley used symbolic language in her novel. 17

100. B A: The author suggests just the opposite. Victor’s father prohibited Victor from pursuing “outmoded, erroneous, semimagical science” (lines 27-29), not modern, mainstream scientific thought. Secondly, there is no indication that Victor’s father held any religious beliefs, or that he would reject “irreligious” scientific ideas. B: Yes. The author describes Alphonse Frankenstein (the father) as a representative of, and believer in, the public or social order (lines 5-13). When Victor pursues his unauthorized studies, he goes against the accepted beliefs and values of that order (lines 23-29 and 60-63). C: While the passage mentions that Victor was imprisoned (lines 10-11), it does not indicate that his father feared or foresaw this possibility. Nor does the passage indicate that Victor’s investigations were actually illegal (or that his investigations were the cause of his imprisonment), only that they were “forbidden” in terms of what was considered acceptable by society. D: While the author of the passage discusses sexuality in her interpretation of the novel, she does not suggest that the characters in the novel themselves saw Victor’s actions and beliefs in a sexual light. Furthermore, Victor’s mother was dead by the time he entered the university (lines 58-59). 101. A A: Yes. The correct answer will be the statement most inconsistent with the passage. The author argues that Victor’s rejection of Elizabeth and his “pursuit” of the monster (in an attempt both to be his mother and to recreate her) indicate that he is breaking with the “normal” public or social realm and is failing to follow the typical Oedipal path (lines 42-53). If Victor instead took the typical step of marrying the “acceptable” mother-substitute Elizabeth, he would stay in that regard within the public realm and follow the “normative” or standard Oedipal path (lines 34-36). B: The author argues that Victor identifies with his mother, and that through creating the monster he attempts to remain with his mother (lines 63-67). If he had nursed his real mother into old age, this would be entirely consistent with the author’s thesis. C: The author’s analysis is based on her interpretation of Victor’s desires and motivations. If Victor failed to bring the monster to life, that is, failed to fulfill his desire, it would have no effect on that analysis. D: The gender of the monster is irrelevant; in fact, the passage never indicates whether the monster in the novel is male or female. If Victor married the monster, it would be entirely consistent with the author’s claim that by choosing it over Elizabeth, the “authorized “ (line 42) or socially acceptable mother-substitute, Victor is leaving behind the public realm for his own isolated imaginary world (lines 46-53). 102. B A: It is unclear from the question whether this refers to the novel’s or the passage’s intended readers. Regardless, the passage does not indicate for whom the novel or the passage was intended, or what those readers’ social attitudes might be. B: Yes. While the author never directly states that Victor’s Oedipal desires are unconscious, throughout the passage she discusses what the characters’ beliefs and actions represent, rather than simply describing those beliefs and actions. This indicates that we are not to take what the characters do or say at face value, but that we should look beneath or beyond their conscious intent. Furthermore, the contrast she presents between the public and private realm comes from the field of psychoanalysis (lines 1-5). As this approach plays a role in all parts of the passage, we can infer that it represents the author’s approach to other novels as well. C: While one socio-historical context [the patriarchal society of Victor’s time (lines 11-13)] is mentioned, this does not characterize the author’s approach to the novel as a whole. It constitutes one reference, not a framework for the author’s analysis. D: As in choice C, this constitutes one reference in the passage (lines 67-70), not a theme that runs throughout the passage.

18

Passage V 103. D A: The passage presents a hierarchy in paragraph 2 based on the cost of different forms of brand placement. We can infer that the most expensive would be the most effective (as there is no reason to think that a company would pay more for a less effective method). Verbal mentions or references are on the list (line 18), but they are only moderately priced. B: Billboards would fall into the category of “simple visual exposure,” which is the least expensive form (lines 17-18), from which we can infer that it would be the least effective method. C: A close-up of the product qualifies as visual exposure, the least expensive, and so we can infer the least effective, form of brand placement (lines 17-18). D: Yes. Character usage is the most expensive type of brand placement (line 19), from which we can infer that it is the most effective (see the explanation for choice A). 104. A A: Yes. In the second paragraph, the author describes the value or benefit of brand placement to both the marketers and the filmmakers (lines 19-26). Later in the passage, the author defends brand placement against its critics by arguing that it does not deceive moviegoers, nor does it force them to buy the products or brands that they see in the movie (lines 73-76). B: See the explanation for choice A. The author clearly speaks as a defender of the practice, not as a neutral observer or analyst. C: This choice is inconsistent with the tone of the passage. While the critics described in the fifth paragraph could be described as mistrustful, the author defends brand placement against those critics (lines 51-53 and 73-76). D: See the explanations for choices A and C. The author has a positive tone, not a disapproving attitude towards brand placement. 105. A A: Yes. This is a paraphrase of lines 7-11. B: First of all, the author states that brand placement “offsets production costs” by providing a source of income to filmmakers (lines 24-26), not that it directly lowers those costs. Secondly, even if this choice were fully supported by the passage, lower cost is not presented as the primary cause of why brand placement is increasing, as described in the first paragraph. C: This is the right answer to the wrong question. That is, it is stated in the passage (lines 24-26), but is not given as the primary reason why the practice is growing. D: While the passage states that this practice has a variety of benefits for marketers, including cost-efficiency and increasing their access to potential consumers (lines 10-11 and 19-24), the author does not suggest that it significantly increases sales, nor that marketers believe that it will. Furthermore, in the last two paragraphs the author suggests that brand placement may have no real effect (lines 53-59) or at most a limited effect (lines 65-73) on product sales. 106. C A: Notice the word “general” in the question. The effect must apply to audiences as a whole, not just to one segment. While young people may get some psychological benefit from the “feelings of belonging, comfort, and security” induced by seeing familiar brands in a movie (lines 34-39), older viewers feel uncomfortable when exposed to the practice (lines 31-34). B: As in choice A, this applies only to one part of the audience: younger viewers (lines 34-36). Older moviegoers experience resistance to and concern about the practice (lines 31-34). C: Yes. While younger viewers may accept the practice as legitimate (lines 34-36), the author claims that all viewers have some resistance to the message. That is, they are not necessarily persuaded to buy the product (lines 70-76). D: This choice is too extreme even as a characterization of older viewers (lines 31-34); the author does not go so far as to suggest that older viewers’ dislike of brand placement keeps them out of the theatres. Furthermore, it is inconsistent with the accepting and even positive reaction of younger patrons (lines 34-39).

19

107. D A: We do not know what the participants were told in this regard. Also, there is no reason to believe that participants’ awareness of a commercial purpose would undermine the credibility of the study. B: As in choice A, there is no basis on which to conclude that age is an invalid distinction to draw, or that socioeconomic status should have been included. In fact, the study may have drawn conclusions based on socioeconomic status that are not discussed in the passage. C: As in choice A, we have no way of knowing that the attitudes of the participants were influenced by others. We also have no basis in the passage on which to conclude that such influence, if it did occur, would undermine the legitimacy of the study. D: Yes. The study concludes that moviegoers are not driven or unduly influenced to buy the products that they see; but only the results of interviews are given (lines 60-65), rather than concrete evidence of their purchasing habits. (It is possible, for example, that people like to think of themselves as independent and skeptical consumers, and so respond accordingly to the interviewer’s questions. They may, however, be more influenced by brand placement than they know or would care to admit.) Therefore, the difference between the content of the conclusion drawn (on actual buying habits) and the evidence for that conclusion (based on people’s impressions or beliefs about their buying habits) is a weakness in the study, based only on information in the passage. 108. A A: Yes. On this question, you must aggressively use process of elimination, and keep in mind that you are looking for the “least wrong” choice. The author states that seeing familiar products in movies gives young viewers warm and fuzzy feelings (lines 37-39). It is somewhat plausible to infer, then, that if there is no increase in Voom sales, viewers may not have experienced those nice feelings when they saw the shoes in the movie. That is, the brand may not be familiar. Now, of course the author never claims that good feelings lead to increased sales. The author even indicates later in the passage that there may be little relationship between brand placement and sales (see the last two paragraphs). However, this is the only one of the four choices that has any connection to information supplied in the passage. B: The passage gives no evidence that would support this claim, nor does it suggest that people will purchase at most one pair of shoes from any one brand. C: The new information in the question tells us that young people make up a large percentage of the film’s audience, and it is young people who are most accepting of, and least troubled by, brand placement (lines 34-39). Older people who might be troubled are not, according to the information in the question, likely to buy Voom shoes regardless of their negative attitudes about product placement. Beyond the issue of brand placement and visibility of Voom shoes, any other social implications these scenes might have are irrelevant to the studies cited in the passage. D: The passage does not suggest any inverse relationship between the audience members’ interest in the movie and their propensity to notice or be influenced by brand placements. Furthermore, in the final paragraph, the author describes movie patrons as “active interpreters, not passive receivers” of brand placements (lines 64-65), indicating that audiences do tend to notice when brands appear. 109. B A: The tone of the passage is clearly partisan, supporting and defending the marketing practices described (see in particular lines 19-26, 51-53, and 65-76). A report from a regulatory agency would presumably include some discussion of potential problems with brand placement and possible regulatory responses to those problems. B: Yes. The passage comes down decisively on the side of marketers and advertisers, promoting the benefits and even necessity of brand placement (lines 7-11 and 19-26), while defending the practice against those who would criticize it (paragraphs five and six). Notice the repeated use of the word “our” (lines 51 and 60). The author is not presenting a neutral description of the positions or opinions of others, but is actively entering into the debate as an interested party. C: The tone and content of the passage indicates just the opposite. A group fighting for consumers’ rights and well-being would likely address an issue only if it were a problem for consumers. This passage, however, attempts to show that brand placement is an innocuous practice involving no deceit or coercion, and that, for the most part, it benefits those involved. It is the critics in paragraph five rather than the author and his or her compatriots who are most likely to be consumer advocates.

20

D: Experimental psychologists would presumably approach the issue from a neutral stance, carrying out scientific, controlled studies to arrive at their conclusions. As discussed in reference to choices A, B, and C, however, this passage is clearly not written by a disinterested observer or analyst. Furthermore, the studies cited and carried out with the participation of the author are based on what appear to be relatively casual, unscientific interviews and group discussions (lines 27-30 and 60-65). 110. B Note: As with Question 108, this question requires an especially aggressive use of POE and a search for the “least wrong” choice, rather than an ideal response. A: The question requires us to accept the author’s conclusions, which include the claim that people are not induced or driven to buy products just because they have seen them in a movie (lines 65-76). The author also argues that people are not only aware of the practice, but also that they have a “sophisticated understanding” of it (lines 34-37 and 60-65). Thus, this answer choice is inconsistent with the author’s conclusions. B: Yes. Given that the author’s claim that brand placement has at the most a slight effect on consumer behavior (lines 65-76), and that we are directed by the question to accept this claim, the primary reason for the increased sales cannot be the appearance of Zog widgets in the movie. Of the four choices, only this one offers an explanation that does not contradict some claim made in the passage, and it could be related to the “perceived needs” mentioned in line 68. C: The author argues that viewers are quite aware of brand placements, and that they are not unduly influenced by them (lines 34-37 and 60-76). The correct answer must be consistent with both the new information in the question and the passage. D: The author describes older moviegoers as resistant to and unsettled by the practice of product placement (lines 31-34). Thus we would expect them to be the least, not most likely to purchase those products. 111. C A: The passage discusses the placement of particular products and brands into movie scenes, not the influence of thematic, noncommercial aspects of the plot and/or characters. B: The author claims that product placement does not cause viewers to run out and buy the products they have seen (lines 73-77). Thus this choice would strengthen, not weaken the arguments made by the author, who writes as a defender of brand placement. C: Yes. Part of the author’s argument in defense of brand placement is that audiences are fully aware of brand placement and that they are not passive objects to be acted upon by marketers (see the last two paragraphs). If a list of props to be seen in the movie changes consumers’ behavior, then it must be telling them something that they did not already know. This would weaken the author’s claim that audiences are already aware of what marketers are doing. D: The passage deals with the appearance of products and brands in movie scenes, not with explicit advertising shown before the movie. 112. D A: This choice is too broad. The author deals with the specific arrangement involved in brand placement, not with business arrangements between marketers and the film industry in general. B: This choice is too narrow. The author does describe some generational differences (lines 30-39), but that is only one small part of the passage. C: This choice contradicts the tone of the passage, which is very positive about the practice of product placement. It is the critics, whose claims the author rejects, who question the ethics of brand placement (lines 49-53). This choice is also inconsistent with the author’s claim in the last paragraph that audiences are fully aware of the practice. According to the author, then, this is not a clandestine, deceptive practice (as would be indicated by the wording “promoting products without seeming to do so”). D: Yes. As discussed in the explanations for Questions 104 and 109, the author presents him or herself as an advocate and defender of brand placement in movies.

21

Passage VI 113. C A: Note the word “major” in the question. The correct answer will describe not only a result or aspect of this “disaster,” but the most significant aspect, in the context of that section of the passage. In lines 6-19, the author describes Henry’s desire to curb or limit the power of the barons through elimination of the local courts over which the barons ruled. The author also describes Henry’s need to make it look as if he were not really imposing radical changes to old practices or to the established rights of those barons. In this context, then, “disaster” would entail rebellion on the part of those whose traditional rights and privileges were in fact being taken away. While this might also involve refusal to participate in the royal courts, the main problem would be resistance to Henry’s attempt to take power away from the barons. B: This choice goes in the wrong direction. The “disaster” would entail resistance to Henry’s attempt to centralize authority by instituting royal courts based on trial by jury, and a resulting movement backward toward baronial authority. This choice describes attempts by citizens to go even further forward by making the juries impartial, which didn’t come about until hundreds of years later (lines 35-41 and 50-54). C: Yes. Henry’s basic motivation in instituting royal courts was to take away some of the power held by the barons or feudal lords (lines 6-8). While doing so, he needed to be careful not to be seen as challenging or taking away the “customary rights” and powers of those barons. The author’s description of the “twenty years of rebellion and anarchy” that preceded Henry’s coronation (lines 6-7) indicates that barons might be predisposed to violently reject the king’s efforts; thus the need to “cloak” his innovations in the guise of old practices, so as not to spark resistance. In this context, the “disaster” would be the failure of this disguise, and a resulting baronial rebellion against the king’s attempt to centralize judicial authority in the hands of the monarchy. D: As in choice A, this might be a side effect of baronial revolt against the royal courts and a return to the old system (or of Henry’s inability to even begin to implement the new courts) (lines 29-31), but it would not be the most significant result or manifestation. Henry’s basic motivation was not to institute a more fair or rational legal system, but to lessen the independence and power of the barons (lines 6-8). See also the explanations for choices A and C. 114. B A: Appeals to higher courts are never mentioned or suggested in the passage. B: Yes. The author draws a contrast between juries in King Henry’s time, which were made up of people actively involved in the case (lines 35-39), and modern juries who ideally know nothing about the incidents involved before they enter the courtroom (lines 45-41). Even hundreds of years later under Tudor rule, jury members were actively involved in the trial as witnesses (and so clearly not impartial), as shown by the fact that they could be charged with perjury, or false testimony (lines 51-54). Therefore, even though the system was evolving towards the impartial or unbiased jury of the modern legal system, that change had not yet fully come about by the time of Tudor rule. C: The passage does hint that at some time between Henry II and the modern era, jury service was not strictly required (lines 42-44), as many of those called failed to show up with no apparent repercussions. Even this, however, does not show that there were no requirements for service, only that some did not comply with any requirements that might have existed. Furthermore, we do not know what requirements for service might have existed as juries evolved through the Tudor era. Most importantly, the author never discusses modern requirements for jury duty. Be careful not to use outside knowledge to answer Verbal Reasoning questions. D: The modern practice of testifying under oath is never discussed in the passage. Oaths are directly mentioned only once in the passage, when the author describes the old system (lines 29-31); we do not know, however, whether or not these oaths were related to testimony. The passage does state that under the Tudor kings jurors could be charged with perjury (lines 52-54), but without additional or outside knowledge, we have no way of knowing if this was based on testimony given under oath (and if it were, it would further invalidate this answer choice).

22

115. D A: While Henry’s motivation in creating the jury system was, according to the passage, the centralization of power, the author suggests that the jury system, once created, contributed to (rather than detracted from) fairness and justice in the English legal system (lines 9-12, 61-64). Therefore, the extension of royal power and safeguards against arbitrary perversions of the law were not inconsistent with each other. B: The author describes medieval juries (in the time of Henry II) as a step away from the arbitrary application of the law which was more likely to occur in the decentralized system that existed previously (lines 9-13). The fact that modern impartial juries may be even less arbitrary and more likely to render a just verdict is consistent, not inconsistent with the assertion cited in the question (lines 55-59 in the passage). C: See the explanation for choice D. The contrast drawn between the old manorial courts and Henry’s jury system is directly relevant. D: Yes. In the last paragraph, the author states that by being judged by “twelve honest citizens” both parties to the case are protected against arbitrary (irrational or biased) applications of the law (lines 55-59). Earlier in the passage, the author describes trial by jury as more “rational” than the “antics” of the old system, which was not based on reasoned analysis and discussion (lines 31-34). Finally (moving backwards through the passage), in paragraph two the author contrasts the consistent and unified “common law” provided by the royal courts and their juries with the disjointed and inconsistent system of the manorial courts (lines 9-14). Thus, the contrast between the irrational and arbitrary manorial system and the rational and consistent jury system clarifies exactly how and why “arbitrary perversion of the law” is less likely to occur under the jury system. 116. A A: Yes. In the last paragraph, the author writes that “the jury system has come to stand for all that is meant by English justice” because of the safeguards provided by an impartial jury of “twelve honest citizens.” The strongly positive tone in this section on modern juries tells us that the author believes that the evolution from manorial courts to biased juries and finally to impartial juries is a very good thing. Thus we can conclude that the author assumes that jurors who have no previous knowledge of the case (lines 39-40) dispense better justice than do those who come to testify as well as to judge (lines 37-39 and 52-54). B: The passage never discusses testifying under oath. See the explanation for Question 114, answer choice D. C: This is the right answer to the wrong question. This statement is supported by the passage (lines 61-64), but it is not directly related to the author’s discussion of changes in the laws of evidence. D: The passage does suggest that modern juries have, through evolution of the laws of evidence, become less biased than juries under the Tudors or in the time of Henry II. However, it does not suggest that the newly created jury system under Henry II was more biased (or that it had different biases) than the manorial courts it replaced. In fact, the passage suggests that even the newly formed jury system was less biased and arbitrary than what came before (lines 9-13). 117. B A: The author would most likely agree with this statement. The author praises the safeguards against arbitrary uses and abuses of the law provided by the jury system (lines 55-61) and appreciates as well the fact that in a jury-based system, it is essentially the people, not the king or centralized power that dispense justice. Thus we can assume that the author would be in favor of constitutional constraints put on rulers and governments. B: Yes. A major theme running throughout the passage is that the evolution of the jury system allows for a more fair and consistent application of the law, compared to the justice dispensed by local magnates or authorities (lines 9-13). The author also indicates that a jury system puts legal power more in the hands of the people than in the hands of authorities (well respected or not), and that this is a good thing (lines 61-64). Therefore, we can infer that the author would disagree with this statement. C: The author never discusses police forces or law enforcement agencies. Therefore, we have no evidence on which to conclude that the author would disagree with this statement. Given the importance placed by the author on protection against the arbitrary or unconstrained application of the law, if anything the author would probably tend to agree. D: The author would most likely agree with this statement, given the passage’s suggestion that the more centralized royal courts were an improvement over the “antics” of the decentralized (local or regional) manorial courts (lines 9-13 and 29-33).

23

Passage VII 118. B A: While this statement is true according to the author (see lines 28-29), it is too narrow to be the major thesis or main point of the entire passage. The main issue for the author is that when self-interested decisions are made by many people on a large scale, it can have a destructive impact on public resources. B: Yes. The author relates the story of the “tragedy of the commons” in which, without regulation, each individual herder acts to maximize his or her own good to the detriment of the community as a whole. He generalizes from this scenario in the last paragraph to argue that we must abandon the “commons assumption.” That is, we cannot allow individuals to use public resources for their own benefit while bringing society to “universal ruin.” His discussion in the first paragraph as well shows that he is speaking in general terms, and that the story of the commons is meant to tell us something about society as a whole. C: This choice is too extreme to be supported by the passage; the author states that coercion is defensible only if a majority agree to it (lines 52-54). Even if it were not too extreme, this choice would be too narrow to be the major thesis, as it is only one specific point made in the fifth paragraph. D: The author uses the metaphor of the tragedy of the commons, not of the invisible hand (which is discussed in the first paragraph) to illustrate what happens to society when individuals act only in accordance with their own self-interest. 119. B A: Remember the main idea of the passage. The author doesn’t indicate that there is a problem with all self-interested actions, only with those that result in the degradation or destruction of public or collectively held resources (see lines 16-19, 38-46, and 59-62). B: Yes. See the explanation for answer choice A. C: While the natural environment (mentioned specifically only in line 12) is one (of many) public or collective resources that should be protected through coercion, the author does not single it out as the decisive determining factor. D: The author does not describe rational self-interested actions that collectively destroy public resources as immoral. For example, acting in accordance with the principle of the invisible hand “[prevents] action based on rational analysis” (lines 4-8); that is, it is unreasonable or illogical or counterproductive, but it is not described as unethical (be careful not to bring your own opinions or beliefs into play). Furthermore, the author never suggests that common beliefs or attitudes are a valid basis for coercion. 120. A Item I:

Item II:

Item III:

Yes. National parks would qualify as a public resource, and mandatory fees would be a potentially acceptable (if agreed to by the majority) form of coercion (like taxes) to keep the parks from being destroyed through overuse (see paragraph five). The author proposes restricting access to public resources in order to preserve them for society as a whole, not taking them away from society as a whole by selling them off to individuals. Don’t forget to use the main idea of the passage for all questions. The passage goes in the opposite direction; the author argues for regulating and restricting, not increasing, individual access to and use of public resources. This choice is inconsistent with the central theme of the passage.

121. A A: Yes. According to the author, rational individuals add up the costs and benefits to themselves [the “component utilities” (lines 31-40)] in order to make decisions, without taking into account the effect on society when all individuals reason in this way. Each billboard benefits the individual manufacturer, while the extent to which a single additional billboard lessens the impact the billboards have on consumers is negligible and is spread out among all who have erected them. However, when all manufacturers act on this logic and the highways are overcrowded with billboards, each individual billboard is very unlikely to be noticed or to have an effect on people driving by (and may even make them angry because they can no longer enjoy the scenery). A few billboards stand out, but hundreds of them blur together. The result is that this form of advertising would then become ineffective for all manufacturers, just as overuse of the commons eventually makes them useless for all of the herders.

24

B: Neither the passage nor the question gives us reason to believe that if lots of graduates accept job offers, there will be some harm to society as a whole, to those graduates, or to future graduates. No public resource is involved in this choice. C: A suburban home is private property, not a public or collectively held resources. There is no indication that this use of private property entails any significant costs to others or to society as a whole, and the ones making the decision (the homeowners) are not the ones who most benefit (the children). D: Emission control standards limit access to public resources (in particular our air and water) so that we can all continue to enjoy those resources in the future. This is the opposite of the logic of the commons. 122. B A: The author presents the tragedy of the commons as a scenario that has already taken place. Notice as well the wording used in lines 48-50, where the author writes that “as the human population has increased, the commons concept has had to be abandoned.” B: Yes. In the case of the commons, no restrictions were placed on how or to what extent individuals used the grazing lands, with the tragic results as described. The author argues that systems of mutual coercion—that is, regulation of some sort—must be imposed to avoid the destruction of public resources (lines 51-58). Therefore, the author would most likely agree that when resources are regulated, self-interested actions will not necessarily drive us to ruin, and thus that the author’s argument would not apply in those cases. C: The author argues that the tragedy of the commons occurs when societies become stable (lines 23-27). This choice would contradict, not clarify the authors’ argument. D: While management might be the same as regulation or coercion, it might not be; management does not necessarily restrict how resources may be used. Furthermore, there is no reason to believe that management which did qualify as coercion—as coercion is discussed in the passage (lines 51-58)—would have to be local to be effective.

Passage VIII 123. B A: The passage does not discuss any individual works of literature. B: Yes. Throughout the passage, the author explains how formalist critics approach literary analysis. In the third paragraph, the passage explains that formalism is concerned only with the work itself, not with how or why that work was written. In the fourth and fifth paragraphs, the passage discusses why formalism is not concerned with the specific characteristics of readers and why formalists assume an “ideal reader,” and then it rejects two different ways of judging a work’s value. Finally, in the last paragraph, the author returns to the idea that a literary work, by formalist standards, cannot be seen simply as the product of the author or through the eyes of its readers. C: While the author does specify which part of the author’s intent is relevant to formalist criticism [that is, what appears in the work itself (lines 31-34)], this is not the same as detailing a method for measuring an author’s intent. Even if it were, this choice would still be too narrow to capture the purpose of the author in the passage as a whole. D: The author does mention that the study of aspects of literature other than the work itself have value (lines 25-30), but not as part of formalist criticism. That is, these undertakings would be defensible as studies, but not as literary criticism, which the author equates with formalist criticism (lines 61-63). In any case, simply describing these alternatives is not the author’s primary purpose in writing the passage. Don’t get trapped by the word “defensible” in line 41; here the author is defending the formalist approach, not alternatives to it.

25

124. C A: This choice is inconsistent with the main idea of the passage. The author rejects the idea that the effect on readers is a relevant focus for formalist criticism (lines 25-30, 34-39, and 61-63). B: This choice is inconsistent with the author’s description of formalist criticism, and so with the main idea of the passage. In lines 19-24, 30-34, and 61-63, the author argues that the formalist critic is not interested in the author’s motivation or intent in writing the work. C: Yes. The author indicates that formalists should focus on “the structure of the thing composed” (lines 22-24), that is, the work itself (lines 19-20; notice the word “primarily” in line 20). This theme—that criticism deals only with the work itself—runs throughout the passage. D: The author argues at the end of the passage that the truth of a work’s ideas, even if such a thing could be measured, does not define good literature. Therefore, we can conclude that the formalist critic would not be concerned with it. 125. B Note: Roman numeral I appears in all four answer choices. Therefore, you do not need to read or evaluate it, as it must be included in the correct answer. Item I: Yes. The author draws this distinction in lines 31-34 in order to emphasize the claim that formalist criticism does not concern itself with the author, but only with the finished product, that is, the work. Item II: Yes. The author draws this distinction in order to show that formalist criticism is not concerned with the readers or the different ways in which a work might be interpreted by various readers. Item III: These two motivations are listed (lines 10-14) as reasons for which an author might write. The passage is not interested in drawing a distinction or contrast between them, but in dismissing them both as irrelevant considerations. 126. A A: Yes. In lines 50-55, the author (writing as a formalist critic—see the word “we” in line 54) argues that neither the sincerity of the author nor the critic’s reaction provides a valid standard by which to measure literary value. This theme continues into the last paragraph, where the author claims that “good literature” is not defined by its causes (the author), its effects (on its readers), or by the truth of its ideas (lines 51-65). Given that the entire passage is a description and defense of formalist criticism, we can assume that the author would consider formalist evaluations to be more valuable. B: This choice contradicts the passage. Formalist criticism dismisses the author’s motivations (lines 10-14, 19-24, 30-34, and 61-63), and the author of the passage believes it to be effective, not ineffective. C: The author never defines the characteristics of an “ideal reader.” In fact, given that this concept is an abstraction used to keep the critic’s focus on the work itself and to eliminate the actual human readers from the equation, we can infer that an “ideal reader” would have no distinguishing features or qualities. D: This is a negation. Take out the word “nothing”, and you have a direct quote from the passage (lines 63-65). 127. C Note: Each answer choice is a quote or paraphrase from the passage. The question asks you to decide which one of these claims is best supported; that is, for which one does the author provide the most evidence? A: The author presents this (lines 14-17) as something that is obviously true, that is, that anyone would obviously know. The author does rephrase it by saying “that is, that they are recreated in the minds of actual readers,” but this does not constitute support for the conclusion itself. B: The author presents this claim as being true by definition. A literary work is a document, and documents can be analyzed in this way, but if we ask why, there is no answer in the passage. C: Yes. The author claims that critics must assume an ideal reader because no other approach is possible. That is, the author claims that this strategy is defensible (lines 39-41), and then goes on to explicitly defend it (lines 4149) by ruling out the three alternatives. D: This claim is asserted as a fact in line 59, with no further evidence or explanation to support it, beyond the statement that no critic would deny its truth (lines 60-61).

26

128. D A: This choice is too extreme. The passage states that psychological studies of literature are “valuable and necessary” (lines 27-29). They just don’t qualify as formalist literary criticism. B: Read the answer choice carefully. It is the formalist critics, not those who apply biographical or psychological studies, who assume that “the relevant part of the author’s intention is what the author actually put into the work” (lines 30-32). C: This choice is too extreme, and it misrepresents the passage in other ways as well. The author states that the analysis of the various readings of a work involves delving into psychology and the “history of taste” (not biography, which is involved in studies focused on the author) (lines 20-22 and 25-28). However, the passage does not also claim the reverse, that is that psychological studies necessarily involve accounting for all possible readings of the work. In fact, some psychological studies of literature focus on the author, not the readers (lines 20-22). Finally, the author rejects a psychological approach to literature not because it might account for multiple readings of a work, but because formalist critics believe that the psychology of authors and readers is irrelevant to the task of analyzing the work itself (lines 1924 and 28-38). D: Yes. In lines 19-24, the author argues that the mind (psychology) and life (biography) of the author are irrelevant; what is important to the critic is the structure of the work itself. 129. C A: The author would disagree with this statement. “The people involved in the literary transaction” would include the author of the work and the work’s readers, including critics. The author of the passage argues that literary criticism— that is, formalist criticism—is not concerned with the life or intent of the author (lines 10-14, 20-24, 30-34, and 61-63), interpretations on the part of various readers (lines 14-20, 25-30, 34-38, and 61-63), or the strength of critics’ reactions (lines 54-55). B: The author would disagree with this claim. The passage claims that the context, historical or otherwise, in which the work was written is largely irrelevant; what matters is the structure of the work itself (lines 19-24, 25-28, and 56-59). C: Yes. This is consistent with the author’s argument in lines 30-34, which asserts that the only aspect of an author’s intent that is relevant to criticism is that which appears in the work. That is, only the finished work itself is important. D: The author never states or suggests that what is taught in English departments has any relevance to a definition of literature. In fact, the passage offers no such definition, on any basis. 130. A A: Yes. By rejecting two nonformalist tests of literary value (lines 49-55), in the context of a passage that offers an extensive explanation and defense of formalist criticism, the author suggests that formalist criticism can in fact evaluate the validity of different literary structures. (In lines 19-24, the author indicates that the focus of formalist criticism is literary structure.) B: Formalist criticism assumes a standardized “ideal reader,” and so does not concern itself with differences between readings of a work (lines 34-49). C: Formalist criticism does not focus on differences between authors (lines 19-24). Therefore, we have no reason to believe that it could be used to distinguish between their personalities. D: The author, a formalist critic (lines 54-55), suggests that there is no way to tell if rhetoric is in fact effective (lines 6668). 131. B Note: The question does not ask which assertion is supported by the passage (they all are), but instead which claim would be most strengthened by the new information in the question. Choose the answer choice which has the most direct relevance to that new information. A: The new information tells us about Hemingway’s own reaction, not the reactions of critics. (The assertion is made in lines 54-55 in the passage.) B: Yes. This would act as a specific example in support of the more general assertion made in the passage that the author’s own feelings tell us nothing about the value of his or her work (lines 52-54). C: The new information in the question makes no reference to Hemingway’s readers. D: The new information in the question has no direct relevance to the author’s discussion of readers’ varied reactions in the passage (in lines 35-36 and 44-47).

27

132. D A: This is stated by the author in lines 17-19. Given that formalist criticism assumes a standardized, abstracted ideal reader (lines 34-41) and so is not concerned with the fact that actual readers’ interpretations will vary, this statement if true would have no impact on the validity of formalist principles. B: The author makes this assertion in the passage (lines 63-65). There is no inconsistency between this claim and the author’s defense of the formalist focus on the structure of the work (lines 19-24). C: The only aspect of the author’s intent with which formalist critics concern themselves is “the author’s intention as realized” in the work (lines 31-34). Therefore, the inability to discern what the author intended to put into the work would pose no problems for formalist criticism. D: Yes. The author explains that formalist criticism looks “primarily at the work itself” (lines 19-20), and indicates throughout the passage that knowledge of the life and mind of the author is not important to an understanding of the author’s work (lines 20-24, 31-34, and 61-63). If it were shown to be true that the social and historical context of a work cannot be ignored by critics, it would undermine the formalist claim that the author, and the author’s own experiences, need not be considered.

Passage IX 133. A A: Yes. If debris from a third ice sheet were found to predate that from the Laurentide and Icelandic sheets, it would support the claim that it was the collapse of that third sheet that caused the changes in the other two, and in the climate as well (see lines 44-47 and 65-68). B: This could still be explained by changes in the Laurentide sheet. The author argues that the collapse of the Icelandic sheet could not be caused by changes in the Laurentide sheet, because the two were not connected (lines 16-22). However, according to the author, a glacier behind the Laurentide could move in synchronicity with it only if it were not a separate sheet (lines 20-23). C: Rafted debris deposited after debris from the Icelandic and Laurentide sheets could not be explained by a third sheet’s icebergs triggering changes that then caused the collapse of the other two. It is much more likely that icebergs from one or both of the other two caused climatic changes which contributed to the collapse of other sheets, which then deposited their debris on top. D: The existence of these currents would indicate that the Laurentide sheet was not the cause of the Heinrich events, but it would also weaken, not support the “third sheet” hypothesis (lines 57-65).

134. A A: Yes. The author states in lines 38-41 that icebergs produced by a surging ice sheet could cause climatic changes by laying down a layer of cold water on top of warm ocean currents. The author indicates that the theory described in this paragraph (based on changes in the Laurentide ice sheet) has been called into question, but not because of doubts about whether or not icebergs from ice sheets can lead to climatic changes (lines 42-47). Later in the passage (lines 57-61), the author also states that icebergs from a third ice sheet could have triggered temperature changes which then caused the collapse, and thus discharge of icebergs, from the Iceland and Laurentide sheets. Therefore, the author would agree that icebergs can both cause and be caused by changes in the climate. B: The author suggests that once Icelandic debris (deposited before or at the same time as debris from the Laurentide sheet) was discovered, it cast doubt on the claim that thickening of the Laurentide sheet caused surging which then created icebergs. Because two separate sheets could not surge on the same cycle (lines 16-22), the Laurentide explanation is called into question, and it was that (now) debunked explanation which gave support to the hypothesis about surging. Therefore, there is no evidence in the passage to suggest that the author would agree with this assertion. C: Volcanoes are mentioned in line 44, but only as a source of debris, not as a cause of surging. D: The studies described by the author use core samples to locate and trace to its origin debris deposited by icebergs, not as direct evidence of climatic change. Furthermore, the author appears to accept the findings of those studies, indicating that he or she would, if anything, disagree with a claim that core samples could not be used as indirect evidence.

28

135. B A: The time frame posited by the passage is 14,000 to 70,000 years ago (lines 25-27). Thus evidence from a million years ago would not be relevant. B: Yes. Bond’s original explanation was called into question by evidence that icebergs from Iceland deposited volcanic debris at the same time or before the Laurentide icebergs (lines 42-55). If that debris were shown to be from Canada, it could have come from the Laurentide sheet [which at times covered North America (lines 6-7)]. Thus Bond’s original explanation (lines 4-11 and 23-41) could still be valid. C: The fact that Earth’s core is cooling would have no effect on Bond’s original hypothesis, which was not based on any particular core temperature or cooling rate (lines 34-36). D: Bond’s original explanation was undermined by evidence of debris from a separate Icelandic ice sheet (lines 42-55). The fact that ocean currents have remained stable over a large part of the 56,000-year period in question would cast doubt on one particular alternative hypothesis—that temperature changes in ocean currents were the real cause of the Heinrich events (lines 62-65). However, it would have no effect on the evidence, or the interpretation of the evidence that cast doubt on the Laurentide explanation in the first place. 136. D A: By providing some additional evidence that icebergs from the Iceland sheet deposited debris at the same time or before the Laurentide icebergs, this finding would decrease, not increase the probability that changes in the Laurentide caused surges (see lines 14-22). B: Evidence of additional debris from the Iceland sheet would have no direct relevance to the possibility that ocean currents were the real cause of collapse. C: Evidence supporting the claim that another ice sheet was collapsing at the same time as the Laurentide would support, not undermine, the probability of an external trigger (lines 14-22 and 50-56). If the Laurentide could not have been the cause, it must have been some other factor. D: Yes. While this evidence supports the claim that there was a trigger external to both the Iceland and Laurentide ice sheets (see the explanation for choice C), it gives no reason to believe that this trigger was a third ice sheet (lines 57-61) rather than ocean currents (lines 62-65), or some other factor not mentioned in the passage. 137. B A: While the author indicates that researchers believe that Heinrich events occur about every 7000 years (lines 2-5), none of their findings or interpretations of evidence are based on this time frame. B: Yes. If the debris layer came from asteroids rather than from ice sheets, it would seriously call into question both the Laurentide explanation and the criticism of that explanation. All of the research described in the passage is predicated on the assumption that the debris was left behind by icebergs discharged from ice sheets. If that assumption is false, the conclusions drawn at first about the action and effects of the Laurentide sheet, and later about the synchronized behavior of the Icelandic sheet, would be without foundation. C: Evidence about how tightly glaciers adhere, and presumably then about how fast they might move, has no direct relevance to any of the research reported in the passage. Glacial movement and friction were factors in Bond’s original explanation (lines 31-38), but as far as we know, he did not claim any particular rate of speed that would be contradicted by this evidence. D: The research in the passage, as described, is not based on the assumption that all cores must show debris. This does not call into question the relevance of samples with debris that have been found, or the conclusions based on those findings.

29

BIOLOGICAL SCIENCES Passage I 138. C. Difficulty in exhaling would lead to an accumulation of CO2, since that is the gas that is exhaled. CO2 is converted by carbonic anhydrase to carbonic acid, which dissociates into a hydrogen ion and a bicarbonate ion; the resulting accumulation of H+ would lead to acidosis (specifically, respiratory acidosis, since it is due to a failure in the respiratory system). O2 would not accumulate, and even if it did, this would not be converted to an acid and would have no effect on pH (choice B is false). The text of the question states that inhalation is not a problem, so there would not be a decrease in O2 intake (choice D is false). Note that choice A, while true, is irrelevant since it does not answer the question. 139. C. Neutrophils are phagocytic white blood cells that typically move to sites of inflammation via chemotaxis. Erythrocytes are red blood cells and do not participate in mediating inflammation (choice A is false), thrombocytes are platelets and play a role in blood clotting, not inflammation (choice B is false), and myocytes are not even a white blood cell—they are muscle cells (choice D is false). 140. B. This sequence is described in the first paragraph of the passage. Choice C also describes events in the proper sequence but lists events that occur several steps after T-lymphocyte activation, making B a better choice. Choices A and D are false; histamine release leads to vasodilation, not vasoconstriction or bronchodilation. 141. A. The second paragraph states, “One therapeutic strategy would be to target a particular subset of T lymphocytes known as T-helper (TH) cells,” and goes on to describe how TH1 cells can inactivate the TH2 cells that are correlated with asthma. Administering IgE may sensitize mast cells, leading to an asthma attack (choice B is false), and neither Zileuton nor colchicine is mentioned in the passage (so choices C and D are false).

Passage II 142. D. Figure 2 shows that Drug C inhibits the binding of cells to both Fg and FN. Choice D is the only choice that shows this. 143. A. Referring again to Figure 2, Drug A inhibits the binding of cells to Fg, but does not affect the binding of cells to FN. This is the same effect as Drug D, described in the question text. 144. C. Figure 1 shows that A5 cells can bind to both Fg and FN (statements I and II are true), but Drug C inhibits binding of A5 cells to FN (statement III is false). 145. B. Since one would not expect water to affect the binding of cells to Fg or FN, and since Drug B does not affect the binding of cells to Fg or FN (choices A and C are false), it seems logical to assume that Drug B might have been water (choice B is true, and D is false). 146. C. Since Drug C affects the binding of cells to both Fg and FN, it seems that this drug would be the one most likely to cause problems. Drug A only affects the binding of cells to Fg (choice A is false), and Drug B has no effect whatsoever (choices B and D are false). 147. D. Based on Figure 2, neither Drug A nor Drug B has an effect on the binding of cells to FN (choices A and B are false), and Drug C decreases the percentage of bound cells (choice D is true, and C is false).

30

Passage III 148. A. From Table 1, we note that the reaction rate is dependent on both the concentration of Compound 1 (Student group 1 vs. 3) and the concentration of sodium ethoxide (Student group 1 vs. 2). The rate-determining step is therefore bimolecular (E2, since this is an elimination reaction). Because E2 reactions involve a transition state in which two s bonds break to form one p bond in a concerted fashion, the students came to the correct conclusion. Br



Br

CH3

H

CH3

H

H

OCH2CH3

OCH2CH3

149. D. The students described the cleavage of two s bonds in a concerted, one-step transition state to form a p bond. This is characteristic of an E2 reaction. 150. C. Compound 1 produces two unique alkenes due to the removal of a b-proton from the methyl group (to produce Compound 3) or from the ring (to produce Compound 2). The only alkyl halide that has two unique b-hydrogen atoms and can therefore produce two alkenes is choice C. Note that choice A would yield a single terminal alkene, while choices B and D do not generate alkenes under these conditions since either the bromine (choice B) or b-hydrogen (choice D) is bonded to an sp2-hybridized carbon. 151. B. Since this is an elimination reaction, the carbon atom bearing the leaving group (bromine) rehybridizes from sp3 to sp2. 152. B. Without a base (NaOCH2CH3) to promote an elimination reaction, Compound 1 will undergo a substitution reaction with ethanol (a nucleophile) to produce choice B. Since the alkyl halide is tertiary, it can ionize to form a relatively stable carbocation, making this substitution reaction unimolecular (SN1).

Passage IV 153. B. These bacteria are described in the passage as producers of toxins that are “superantigens”; i.e., antigens that produce an exaggerated immune response. The passage does not discuss their reproduction (choice A is false) or the inhibition of metabolic enzymes (choice D is false). Autoimmune reactions, by definition, are abnormal immune reactions to normal cellular antigens, not to bacterial antigens (choice C is false). 154. B. The primary structure of proteins is their amino acid sequence. The secondary structure of proteins involves interactions between the backbone of the peptide chain to form structures such as a-helices and b-sheets (choice C is false), and the tertiary structure of proteins involves interactions between amino acid side chains to form the overall three dimensional structure of the protein (choice D is false). The isoelectric point of amino acids has nothing to do with their sequence in a particular protein (choice A is false). 155. C. The passage states that TSS is characterized by a high fever, hypotension (circulatory system), and a rash (skin). Fever is a non-specific disease defense mechanism, and of the four systems listed as answer choices, the lymphatic system is most directly involved with disease defense. 156. C. The passage states the conventional antigenic stimulation activates 1 in 100,000 T lymphocytes, or 0.001%. Superantigens activate 20% of T lymphocytes, or 20,000 of 100,000 cells. 157. D. If the dose of Strain A required for infection is greater than the dose of Strain B required for infection, then Strain A must be less potent than Strain B (choices A and C are false). More specifically, since the dose of Strain A required for infection is twice the dose of Strain B required for infection, Strain A must be half as potent as Strain B (choice D is true, and B is false). 31

Independent Questions 158. A. Adding a repressor protein that binds to the superantigen gene would prevent its transcription (and its subsequent translation; choice B could prevent synthesis and can be eliminated). Adding a complementary nucleic acid sequence that can bind to superantigen mRNA would prevent its translation (choice C could prevent synthesis and can be eliminated). Adding a stop codon within the superantigen gene would result in a stop codon in the superantigen mRNA upon transcription. This would terminate protein synthesis prematurely when the mRNA was translated (choice D could prevent synthesis of the complete protein and can be eliminated). However, tRNA nucleotides that bind to both mRNA and ribosomes are normal—this is how mRNA is translated. Adding more of these nucleotides would not prevent (and may enhance!) translation of the superantigen protein (choice A would not prevent synthesis and is the correct answer choice). 159. B. The O at position a is the same as the corresponding O in the hemiacetal, so choice A is eliminated. The hydroxyl group in the hemiacetal is released and replaced by the OCH3 (methoxy) group of methanol. So, the oxygen at position b comes from the 18O labeled methanol, while the oxygen at position c comes from the released hydroxyl group. 160. D. The terminal amine of phenylhydrazine (recall that hydrazine is H2NNH2) will react with the carbonyl shown in the question (similar to imine formation). The first step of this addition–elimination reaction is addition. H+ O

CH3

HO

CH3 OH

CH3

addition HN

elimination

N

NHPh

NHPh

H2N NHPh phenylhydrazine

161. D. Parthenogenesis (choice D) is the activation and subsequent development of an egg without the use of a sperm, for example by pricking it with a fine needle. Isogamy (choice A) is a condition in which male and female gametes are visibly indistinguishable, but this does not preclude their ability to join in sexual reproduction. Hermaphroditism (choice B) and pseudohermaphroditism (choice C) are states in which a single animal produces both sperm and eggs and is able to selffertilize; although unusual, they are examples of sexual reproduction. 162. C. If there are 12 nucleotide base pairs for every complete 360∞ turn of the helix, there must be one base pair for every 30∞ of the turn. This most closely corresponds with the data for the Z conformation of DNA given in the question text.

Passage V 163. C. As stated in the passage, isoprene is a 5-carbon unit; two isoprene groups form a terpene. Cembrene (Compound 7) has 20 carbon atoms, making it a diterpene (20 carbon atoms = 4 isoprene units = 2 terpene units). b-Elemene (Compound 9) has 15 carbon atoms (3 isoprene units = 1.5 terpene units), making it a sesquiterpene. 164. C. Conjugated p bonds produce strong UV absorptions. While Compound 5 has three double bonds, they are not conjugated, and will therefore not produce a strong UV absorption (eliminating choice A). While hydroxyl groups (choice B) and carbonyl groups (choice D) are prominent in IR spectroscopy, they do not produce strong UV absorptions. 165. A. (–)-b-Elemene (Compound 9) is shown in Figure 1 in the passage. Its enantiomer, (+)-b-elemene, will have the opposite stereochemistry at all three stereocenters. Note that choices B and D have only two of the three stereocenters inverted, and choice C has only one.

32

166. D. Since the procedure must convert an ester functional group (–OAc) into an alcohol functional group (–OH), choices A and C are eliminated, and condensation (choice B) does not apply here. The answer is D: Saponification is the hydrolysis of an ester using a base. 167. A. In thin-layer chromatography (TLC), the stationary phase is highly polar, while the mobile phase is less polar. Compounds that are more polar therefore have greater affinity for the stationary phase and a slower migration. Compound 5, with its two hydroxyl groups, is the most polar of the four compounds listed in the answer choices and will therefore migrate the slowest.

Passage VI 168. A. Referring to Figure 2, it is clear that since the gene is being expressed in some tissues and not in others, it is not “spreading widely throughout the fetus” (choices C and D are false). Furthermore, since some of the organs in which the gene is expressed are large (such as the liver) and some are smaller (such as the pancreas), it is not “only expressed in the largest organs” (choice B is false). However, all of the organs in which the gene is expressed are abdominal organs, making choice A the best choice. 169. D. The passage describes the cationic lipids as being positively charged, so DNA must be negatively charged (choices A and B are false). Furthermore, the whole reason the DNA must be coated with lipids in the first place is because it is too hydrophilic to easily cross the cell membrane on its own (choice D is true, and C is false). 170. B. The cell membrane does not contain DNA. In eukaryotes, DNA is found in the nucleus, and in prokaryotes it is found in the cytoplasm (choices C and D are false). Furthermore, the cell membrane is not a pure lipid bilayer; it contains proteins in addition to phospholipids, cholesterol, and carbohydrates (choice B is correct, and A is false). 171. C. The only cells that are passed from parent to offspring are the germ line cells (eggs and sperm). All other cells are somatic cells and are not passed on to the next generation; thus, a gene introduced to a somatic cell could not be passed on to offspring (choice C is true, and choices A and B are false). Furthermore, any DNA contained within a germ line cell, be it original cellular DNA or introduced DNA, will be passed on (choice D is false). 172. A. All cells in a multicellular organism contain the same genome, inherited from the original egg and sperm that fused during fertilization (choices C and D are false). What makes one type of cell different from another type of cell is not the DNA it contains, but the specific genes within that DNA that are or are not expressed. Since all cells do not express the same genes, cells can vary widely in their appearance and function (choice A is true, and B is false).

Passage VII 173. A. The passage states that fighting big fires is a frightening experience. Frightening and/or stressful experiences activate the sympathetic division of the autonomic nervous system, which, among other effects, acts to increase the heart rate (choices C and D are false). Since the passage goes on to state that Steve’s experience was especially frightening, one can assume that his heart rate would be more affected than Joe’s (choice A is true, and B is false). 174. C. Since Joe has suffered extreme skin scarring (as described in the passage), he would have more difficulty cooling by sweating than Steve. Older males have lower metabolic rates than younger males and would therefore tend to have lower body temperatures (choice A is false), and if Steve has a greater percentage of body fat than Joe, Steve would tend to overheat more easily than Joe (choice B is false). Furthermore, if Steve weighs half as much as Joe (as stated in the passage), then he has a greater surface-to-volume ratio than Joe (choice D is false). 175. C. The oxygenation of blood can be affected by many things, including the rate and depth of breathing (increases in these values can lead to greater oxygenation, choice A is false), hemoglobin concentration (increases in [Hb] can increase oxygenation, choice B is false), and alveolar surface area (decreases in surface area, such as in emphysema, can decrease oxygenation, choice D is false). However, blood pressure in the pulmonary artery is unlikely to affect oxygenation, making C the best answer choice. (Note, however, that if blood pressure dropped dramatically, oxygenation would be affected, but of the four choices given, this is the least likely to have an effect.) 33

176. D. Since glomerular filtration rate (GFR) is directly proportional to blood pressure, increases in blood pressure would lead to increases in GFR. Therefore, if Steve’s blood pressure increased more than Joe’s, one would expect Steve’s GFR to increase more than Joe’s (choice D is true, and A is false). Reabsorption rate is not affected by blood pressure (choices B and C are false). 177. C. Since Joe weighs twice as much as Steve, he would require more energy to perform the same work tasks (choice C is true and A is false). Choice B is contradictory; an individual with a greater basal metabolic rate would not consume less energy. Choice D is irrelevant, since the question asks about the energy involved in performing work, not the energy involved in maintaining the body at rest (basal metabolic rate). 178. D. Since the liver is not involved in the production of digestive enzymes (choice A, the job of the pancreas), antidiuretic hormone (choice B, the job of the hypothalamus), or new blood cells (choice C, the job of the bone marrow), damage to the liver should not affect their production. However, since the liver is directly involved in the production of bile salts, liver damage should reduce their production. 179. B. The blood pressure of the firefighters is given in the passage as 125/70 mm Hg; this represents the systolic blood pressure over the diastolic blood pressure. The systolic blood pressure (125 mm Hg) is the pressure measured when the heart is contracted, and the diastolic blood pressure (70 mm Hg) is the pressure measured when the heart is relaxed (choice A is false, and B is true). Note that both choices C and D refer to the pressure when the blood is entering an artery (in other words, when the heart is contracting), thus these measurements must be systolic measurements (choices C and D are false).

Passage VIII 180. A. The aldol/decarboxylation reaction will begin (in the presence of a base) with the deprotonation of the diacid to produce a nucleophilic enolate. The primary function of pyridine is to facilitate this deprotonation (choice A). [Note that pyridine will also deprotonate the carboxylic acid functional groups of the diacid (choice B), but this does not result in their neutralization, as the answer choice says.] 181. B. SOCl2 converts carboxylic acids into acid chlorides (and alcohols into alkyl chlorides). 182. C. Since the methyl groups replace all four carbonyl a-protons, enolization is not possible (choice C). 183. D. By analogy to the passage, methyl propenoate would be converted to 1-propanol: OCH3

OCH3

H2/Pd/C condition d

LiAlH4

OH

condition e

O

O

methyl propenoate

methyl propanoate

1-propanol

Condition d results in the reduction of the double bond, while condition e (followed by an acidic workup) reduces the ester to an alcohol.

34

184. B. The passage states that Compound 6 was reacted with 2-methylpropanoic acid in the presence of the strong base lithium diisopropylamide (LDA). LDA deprotonates 2-methylpropanoic acid (first at the carboxylic acid, then at the aproton) to generate an enolate. This enolate then reacts with Compound 6 to give choice B:

CO2–Li+ OH

LDA

O 2-methylpropanoic acid

O–Li+

1 equivalent Compound 6

H+

choice B

O–Li+ 2 equivalents

CO2–Li+

Independent Questions 185. D. The function of antidiuretic hormone (ADH) is to allow the kidney tubules to reabsorb more water, thus retaining water in the body. After being dehydrated for two days, the hiker’s ADH levels would be expected to be high. Glucocorticoids (choice A) have various jobs, including reducing inflammation and raising blood glucose during stress, aldosterone (choice B) increases Na+ retention, and insulin (choice C) lowers blood glucose. 186. A. Endocytosis is the uptake of extracellular material. Phagocytic cells of the immune system, such as macrophages (choice A) and neutrophils, would be expected to have the highest rates of endocytosis. Erythrocytes (choice B) are red blood cells and function in oxygen transport, osteoblasts (choice C) are bone-building cells, and neurons (choice D) send nerve impulses between the brain, spinal cord, and body organs. 187. C. Antibody proteins are synthesized by B-cells and secreted into the blood. Secreted proteins are translated by ribosomes attached to the endoplasmic reticulum (specifically, the rough ER). The nucleus and mitochondria are not associated with protein translation (choices A and B are wrong), and while the Golgi apparatus is involved in the secretory pathway (the proteins would interact with the Golgi), it is not specifically involved in the translation (C is a better answer choice than D). 188. C. Transmission at the neuromuscular junction (NMJ) is chemical in nature. Neurons release ACh onto muscle cells; the ACh binds to receptors on the muscle cells and triggers their depolarization (and thus their contraction). The ACh is then degraded by a cholinesterase found at the NMJ. Addition of a cholinesterase blocker would prevent the degradation of ACh, thereby prolonging the stimulation of the muscle and preventing it from repolarizing in order to receive another impulse (choice A is true and eliminated). Addition of a toxin that prevents the release of ACh would certainly interfere with impulse transmission; if the neurotransmitter can’t be released, the muscle cells cannot be stimulated (choice B is true and eliminated). Addition of a substance that binds to ACh receptors would prevent ACh from binding and prevent impulse transmission to the muscle cell (choice D is true and eliminated). The only choice that would not interfere with transmission is choice C; an increase in ACh receptor sites would enhance impulse transmission. 189. C. The fact that the lens fails to form in the absence of the optic cup indicates that the optic cup is necessary for lens development, and may in fact induce it. This says nothing about the timing of neurulation relative to gastrulation (choice A is false), or about the specific timing of eye development (choice B is false). Choice D is a false statement in addition to being irrelevant; cell differentiation is a gradual process, not an all-or-none event, and in any case, this says nothing about the relationship between optic cup development and lens development.

35

Passage IX 190. A. The passage states that if the Xi genotypes are not selected against, the frequency of Xi should increase to 100%. Thus all males would have the XiY genotype and sire only daughters, and the population would become increasingly female (in other words, not stable, choices B, C, and D are false). Ultimately, the entire population would be female and would become extinct. 191. A. The last paragraph of the passage gives a clue that the affected process might be meiosis, since the XiY males are apparently defective in properly segregating their sex chromosomes. Further, in the second paragraph the passage states that the total reproductive output of the XiY males is not affected. If mate recognition (choice B), courtship behavior (choice C), or genital development (choice D) were disrupted, total reproductive output would be reduced (choices B, C, and D are false, and choice A is true). 192. C. The words “random mating” are a tip-off for Hardy–Weinberg genetics. Since all males receive a Y chromosome from their fathers, the male contribution can be ignored; we are only interested in the type of chromosome the females will donate. Let us arbitrarily assign Xi as p in the H–W equations, and Xs as q. 15% of the females are XiXi (15% pp), 50% are XiXs (50% pq), and 35% are XsXs (35% qq). Plugging these values into the H–W equation for genotype frequency (pp + 2pq + qq = 1) gives us 0.15 + 0.50 + 0.35 = 1; solving for p and q gives us approximate values of p = 0.4 and q = 0.6. Since H–W equilibrium requires that allele frequency remain the same from generation to generation, and since males can only inherit either Xi (p) or Xs (q), the proportion of XiY males is simply the frequency of the Xi allele: p = 0.4 = 40%. 193. D. The passage states that none of the Xi genotypes are selected against; therefore, no Xi genotype can have greater fitness than another (choice A is false). There is nothing in the passage to suggest that XiXi flies are more likely to migrate than any other genotype (choice B is false). Since females have only X chromosomes to donate, they have no choice but to pass their X to all their offspring, and there is nothing in the passage to suggest otherwise (choice C is false). Choice D is true; since XiY males sire only daughters, all their offspring receive their X. However since XsY males sire equal numbers of daughters and sons, approximately half their offspring receive an X and half receive a Y. 194. A. Since the original female produced both sons and daughters, the male she mated with must not have been XiY (choices B and D are false). Furthermore, since some of her sons sire only females, and some of her sons sire both males and females, she must be XiXs; sometimes she passes on Xi to her sons, and sometimes she passes on Xs (choice C is false). 195. C. The characteristics of the occasional sons of XiY males are that they are “viable” (choices A and B are false), “sterile” (choice C is true), and “of normal appearance” (choice D is false). 196. B. The passage states that the Xi frequency will reach 100% if all genotypes have equal fitness (are not selected against). If we want to prevent Xi frequency from reaching 100%, then all genotypes must not have equal fitness (choices C and D are false). In other words, some genotypes must have greater fitness than others; if the balance is to be swayed toward Xs, then these genotypes must have the greatest fitness (choice B is true, and A is false).

Passage X 197. A. This question is difficult because it is hard to decipher precisely what the question is asking. The best bet is to eliminate incorrect or irrelevant choices, based on the theme of the passage. The main theme here is that altered expression of particular genes during development, or possibly mistakes in transcript processing, could lead to disease. There is no reason to assume that certain genes are better “targets” for mutation than other genes, so choice C is false. Choice D may be true, but does not fit the “theme” of the passage, so choice D is eliminated. Both choices A and B are reasonable answers, but since the passage specifically discusses how gene expression is altered in this particular disease (NF1), choice A is a better response than choice B.

36

198. D. The passage describes this disease as having a wide variety of clinical features, including skeletal abnormalities, learning disabilities, and tumors. Clearly these clinical manifestations do not affect the same tissues (choice A is false), and they do not arise from the same cell type (choice B is false). The passage says that the disease could arise from a single genetic defect, but that the wide variety of symptoms makes this unlikely. Furthermore, the entire point of the passage is that altered expression of genes can lead to a variety of protein products from a single gene sequence, and that abnormal expression can lead to disease. This makes choice D better than choice C. 199. A. Changes in a gene’s transcript (i.e., its mRNA) will result in changes in the protein product, not in the gene itself (choice D is false) or in its transmission (choice C is false). Protein is translated (synthesized) by ribosomes, not transcribed by ribosomes (choice A is true, and B is false). 200. B. The mechanisms that regulate gene expression are not simple; in fact, this is stated in the passage in the last paragraph (choices A and D are false). Further, this passage describes differences in mRNA transcripts, so the mechanisms cannot affect only DNA (choice B is true, and C is false). 201. C. One of the main reasons that differentiated cells are different from one another, in both structure and function, is that they produce different proteins. Pancreatic cells, for example, produce insulin but not acetylcholine; neurons produce ACh but not insulin. This can explain why different transcripts are needed as cells differentiate. Note that although choices A, B, and D all represent true statements, they fail to answer the question; they do not explain the need for different transcripts, making choice C the best option. 202. A. Epigenetic (developmental) modulation of gene expression is important in evolutionary terms, because it allows the expression of multiple types of protein from the same basic DNA sequence. In other words, it allows greater variability within an organism (and thus possibly greater fitness within the population); choice A is true. Choice B may be a true statement, but this is not epigenetic modulation as described in the passage (choice B is eliminated). Posttranslational modification of defective proteins will have a limited function in evolution at best, since proteins are not passed down from generation to generation. It is much more likely that modification of functional genes (that can be passed down to the next generation) would play a greater role (choice C is false). Choice D is false because, again, epigenetic modulation of gene expression involves changes in RNA, not variability in DNA. 203. D. If NF1 is caused by a dominant allele, then individuals who are homozygous dominant (DD) or heterozygous (Dd) could display this disorder. Choices A and C are characteristic of recessive disorders (in which the homozygous recessive genotype is necessary for the disease to be expressed), and choice B is not true of any disease-causing allele.

Passage XI 204. C. Since the alkoxide used in Student 2’s proposal is t-butoxide, t-butanol should be used as the solvent. This will prevent the deprotonation of solvent by t-butoxide from forming any undesired products. For example, if methanol were used as a solvent, dimethyl ester could be produced: (CH3)3CO–K+

CH3OH

(CH3)3COH + CH3O–K+

CH3X

CH3OCH3

205. A. There are 2 groups of nonequivalent protons in the desired product, in a ratio of 3:9 (or 1:3). b CH3 a CH3O

C

b CH3

CH3 b

Neither of these groups has any neighboring protons, so they will both appear as singlet peaks in the 1H NMR spectrum (eliminating choices C and D). Since (the protons in) Group a are closer to the electron-withdrawing oxygen atom, they will appear farther downfield, eliminating choice B. 37

206. B. The reaction shown in the second step of Student 2’s scheme is a substitution reaction (eliminating choices C and D), since one s bond is broken and one is formed. Since the electrophile is a primary alkyl halide, it will not form a carbocation, and the reaction must proceed by an SN2 mechanism. 207. D. The aromatic halide Ar3CX (“Ar” = aryl or aromatic; for example, Ph3CX) is not only a tertiary halide (favoring carbocation formation), but the resultant carbocation is stabilized by resonance with the three aromatic rings. Therefore, Ar3CX readily forms a carbocation to undergo SN1 reactions with alcohols. 208. A. The reaction of a general dihalide, R2CX2, with two equivalents of alkoxide will produce an acetal, R2C(OR)2.

Independent Questions 209. D. The distal tubule and collecting duct have variable water permeability based on the presence of ADH; in its presence they are water-permeable (allowing increased water retention by the body), and in its absence they are waterimpermeable (allowing increased water elimination in the urine). The glomerulus (choice A) is the site of blood filtration, Bowman’s capsule (choice B) is the tissue surrounding the glomerulus, and the loop of Henle (choice C) is a site of water reabsorption, but it is not affected by ADH. 210. D. Since the disease is more prevalent in males than in females, it must be sex-linked (choices A and B are false), and since it skips generations (in most cases), it is most likely a recessive disorder (choice C is false). 211. B. It is clear from the question text that an attempt to run glycolysis is being made. However, glycolysis requires an input of two ATP molecules before any ATP or lactic acid can be produced. O2 (choice A) is not required; the text states that this is an anaerobic experiment. NADH (choice C) is a product of glycolysis, not a substrate. Acetyl-CoA (choice D) is produced from pyruvate under aerobic conditions, and again, is not a substrate in glycolysis. 212. D. Recall that bromine reacts with alkenes to produce dihalides, a reaction which decolorizes bromine. Based on the table given, tung oil contains the most double bonds, since its hydrolysis results in primarily dienoic and trienoic acids. Tung oil will therefore decolorize the largest volume of bromine. 213. B. Epinephrine is released from the adrenal medulla under sympathetic stimulation and is not regulated by the anterior pituitary at all. The release of cortisone (choice A), progesterone (choice C), and thyroxin (choice D) are controlled by ACTH, LH, and TSH, respectively, released from the anterior pituitary. 214. C. Bone consists of calcium phosphate crystals deposited on a collagen matrix (choices A and B are false). Collagen has a unique triple-helix structure that depends on the presence of hydroxylysine and hydroxyproline residues for stability (choice D is false). The only substance listed here that is not required in bone is potassium (choice C).

38

View more...

Comments

Copyright ©2017 KUPDF Inc.
SUPPORT KUPDF